Optics Assign PDF

You might also like

Download as pdf
Download as pdf
You are on page 1of 115
Exercis: OBJECTIVE PROBLEMS (JEE MAIN) SECTION (A): PLANE MIRROR Point source of light is placed in front of a plane mirrox, (A) Only the reflected rays close to the normal meet at a point when produced backward. (B) All the reflected rays meet at a point when produced backward. (C) Only the reflected rays making @ small angle with the mirror, meet at a point when produced backward. (0) Ught of different colours make different images. Sol. 2. A point object is kept in front of a plane mirror. The plane mirror is doing SHM of amplitude 2cm. The plane ‘mirror moves along the x-axis and x-axis is normal to the mirror. The amplitude of the mirrors such that the objectis always infront of the mirror: The amplitude’ of SHM of the image is (Ayzero (8) 2am Sol. (4am (0) 1m 3. A watch shows the time as 3 : 25. What will be the {Ume that appears when seen through a plane mirror ? (8235 (8)9:35 (C)7:35 (08:25 4. If a ray of light is incident on a plane mirror at an angle 60° from the mirror surface, then deviation Produced by mirror is. (A) 30° (B) 60" Sot. (E90% (0) 1208 '5. When light is reflected from a mirror a change occurs in its : (A) phase, (8), frequency, (C) wavelength, (0) speed Sol, GeThe images of douds and trees in water always ess:bright than in reality ~ (A) because water Is forming the image dirty (B) because there is an optical illusion due to Which the image appears to be less bright (C) because only a portion of the incident light is reflected and quite a large portion goes mid water (D) because air above the surface of water contains jot of moisture Sol, 7. A rays is incident at an angle 38° on a mirror, The angle between normal and reflected ray is (A) 38° (8) 52° (c) 90° (0) 76° Sob, & Mark the correct options. (A) If the incident rays are converging, we have a real object. (8) If the final rays are converging, we have a rest image. (C) The image of a virtual object is called a virtual image. (0) If the image is virtual, the corresponding object is called a virtual object. Sol. A point source of light is placed in front of a lane mirror, (A) All the reflected rays meet at a point when Produced backward. (B) Only the reflected rays dose to the normal ‘meet at 2 point when produced backward. (C) Only the reflected rays making a small angle with the mirror, meet at a point when produced backward. (0) Light of different colours make different images!” Sol. 10. Which of the following letters do not surface lateral inversion (A)HGA (8) HOX Sol. (c) veT —(D) YUL 11, An object is initially at a distance of 100 cm from 2 plane mirror. If the mirror approaches the object at'a speed of S cm/s. Then after 6 s the distance between the object and its image will be : (A) 60 cm (8) 140 cm (©) 170 cm (D) 150 cm Sol. 12. Two mirrors are placed perpendicular to each other. A ray strikes the first mirror and after reflection from the first mirror it falls on the second mirror. The ray after reflection from second mirror will emerge: (A) Perpendicular to the original ray (B) Parallel to the original ray (C) At 45° to the original ray (0) At 60° to the original ray Sol. 13. A person is in ajroom-whose ceiling and two adjacent walls are mirrors. How many images are formed ? A) 5 Sol. (es 7 (08 14, If an object is placed unsymmetrically between two plane mirrors, inclined at the angle of 60°, then the total number of images formed is A) S (8) 4 (C) 2 (0) infinite Sol. SECTION (B): SPHERICAL MIRROR 15. In image formation from spherical mirrors, only paraxial rays are considered because they : (A) are easy to handle geometrically (8) contain most of the intensity of the incident light (C) form nearty a point image of a point source (0) show minimum dispersion effec. Sal. 16. A concave mirror of radius of curvature 20 cm forms image of the sun. The diameter of the sun subtends an angle 1* on the earth. Then the diameter of the image is (in cm) : (A)2v9 (8) 1/9 (20 (0) 18 17. A convex mirror has a focallength /. A real object, placed at a distance / in front of it from the pole, produces an image at (ar (2 or (0)= ‘Sol. 18. A convex mirror has a focal length = 20 cm. A ‘convergent beam tending to converge to a point 20, ‘am behind convex mirror on principal axis falls on it. ‘The image if formed at (Ayintinity (8) 40.cm (€)20em — (D) 10am 19. An object is placed ata distance u from a concave mirror and its real image is received on a screen placed at a distance of v from the mirror. If f is the focal length of the mirror, then the graph between 1/v versus, ais 1 v v “ @) A i 7 3 q ‘ ©. ©) 1 4 Sol . 20, Aninfinitely long rod Wes along the axis of a concave ‘mirror of focal length f. The near end of the rod isat 2 distance u > f from the mirror. Its image will have a length Fu -f) (8) utu-) (Crust (Dy ut +A Sol. 24, Acandle is kept at a distance equal to double the focal length from the pole of @ convex mirror. its magnification will be : (0)-2/3 @-73 (8)3 (C23 Sol. 22, A concave mirror gives an image three times as large as the object placed at a distance of 20 cm from it. For the image to be real, the focal length should be : (A) 10 em Sol. (8)1Sem — (C)20em (0) 30em 23. If an object is 30 cm away from a concave mirror cof focal length 15 cm, the image will be (A) erect (8) virtual (©) diminished (0) of same size ‘Sol. 124. A concave mirror cannot form : (A) virtual image of virtual object (8) virtual image of a real object (C) real image of a real object (0) real image of a virutal object Sob 25. The largest distance of the image of a real object from a convex mirror af focal length 20 cm can be : (A)20cm —(B) infinite (C) 10.6m, (©) depends on the postion of the object. ‘Sol, 26. A partide is moving towards 2 fixed spherical mirror. The image (A) must move away from the mirror (B) must move towards the mirror. (C) may move towards the mirror. {D) will move towards the mirror, only if the mirror is, convex, Sol. 127, A straight line joining the object point and image point is always perpendicular to the mirror (A) if mirrors plane onty (B) if mirror is concave onty (C) if mirror ts convex onty (D) none of these Sol. 28, The focal length of spherical mirror is (A) Maximum for red light (8) Maximum for blue light (C) Maximum for white light (D) Same for all lights Sol, 29. A virtual image, larger. than the object) can be produced by (A) convex mirror, (C) plane mirror Sol. (8) concave mirror (D) concave lens 30, In case of concave mirror, the minimum distance between a real object and its real Image is : war (8) 2F (oar (D) zero Sol. 31, The rear-view mirror of a car is : (A) Plane (8) Convex (©) Concave (0) None of the above ‘Sob. 32. A candle flame of 3 cm is placed at 300 cm from 2 wall. A concave mirror is kept at distance x from the wall in such a way that image of the flame on the wall is 9 cm. Then x is = (A) 339 em (B) 900 cm (©) 450 cm (0) 423 cm Sol, ‘SECTION (C) : REFRACTION IN GENERAL, REFRACTION AT PLANE SURFACE ANDT.LR. 33. When a wave is refracted : (A) its path must change (B) its amplitude must change (C) its velocity must change (0) its frequency must change ‘Sol 34. A ray incident at a point at an angle of incidence of 60° enters a glass sphere of = v3 and its reflected and refracted at the farther surface of the sphere. ‘The angle between reflected and refracted rays at this surface is (a) 50° (040° Sob (8) 90° (c) 60° 35. A,ray of light passes through a plane glass slab of thickness t and refractive index = 1.5. The angle between incident ray and emergent ray will be (0) 60° (a)or (8) 30° (case Sol. 36. A beam of light Is converging towards a point. A plane parallel plate Of glass of thickness t refractive index ws is Introduced in the path of the. beam. The convergent point is shifted by (assume nede normal incidence) : (a) (1-2 away (0) {142 away ar Uy (0) {1+ reo (s1 © 11- jpsarer Sol, 137. A ray of light 1s incident on one face of 2 transparent slab of thickness 15 cm. The angle of inadence is 60°. If the lateral displacement of the ray ‘on emerging from the parallel plane is 5¥3 om, the refractive index of the material of the slab is. (A)1.414 — (B)1.532 (C)1.732 (0) none ‘Sol, 38. The critical angle of light going from medium Ato ‘medium Bis 0. The speed of light in medium A is v. The speed of light in medium B is : Oo Sob (8)vsine (C)vcoto (0) vtano 39. Aray of light from a denser medium strike a rarer ‘medium. The angle of reflection is r and that of refraction is r, The reflected and refracted rays make ‘an angle of 90° with each other. The critical angle witl be A) sin (tan e) (C) sin" (tan) Sol. (8) tan* (sin r) (0) tan" (sin r}) ‘40. Two transparent media A and B are separated by 2 plane boundary. The speed of light in medium A is 2.0 x 10*m s"! and in medium Bis 2.5 x 10* ms". The critical angle for which array of light going from A to B istotally internally reflected is cvs sre(5} car (3) cer a" (5) or (5) 44, A small source of light Is 4m below the surface of a liquid of refractive index 5/3. In order to cut off all ‘the light coming out of liquid surface, minimum diameter of the disc placed on the surface of liquid is (a)3m (8) 4m (C) 6m (D)= Sol. ‘42. A fish looking up through the water sees the out- side world contained in a circular horizon. If the re= fractive index of water is 4/3 and fish is.12.cm below the surface, the radius of the circle!in emis, (a) 12 «3 x (8)4* 5 (12 *3* FG (0)12%3/.7 ‘Sol. ‘SECTION (D) : REFRACTION BY PRISM 43. A ray of light is incident at angle / on a surface of 2 prism of small angle A 8 emerges normally from the ‘opposite surface. If the refractive index of the material of the prism Is n, the angle of incidence / Is nearly equal to : A A Oy (ye A 8) Gy (CWA Tt produces a deviation of 44° when the angle of incidence is either 42° or 62°. What is the angle of incidence when it is undergoing minimum deviation? (ayas® ()ss* Sot. (B)49" (40% 4S. Array incident at angle 53° on a prism energes at {an angle at 37° as shown. If the angle of incidence is, ‘made 50*, which of the following is a possible value of the angle of emergence. (a) 35° (cy4oe (0) 38° (e) 42° 46. A beam of monochromatic light is incident at |= 50° on one face of an equilateral prism, the angle of ‘emergence s 40°, then the angle of minimum deviation is: (A) 30° ‘Sot. (8)< 30° (C)<30" (0) >30° 47. A prism has a refractive index > and refracting langle 90°, Find the minimum deviation produced by prism. (ay4oe (8) 45° (C) 30° (0) 49° £ 48. A prism is made up of material of refractive index JB. The angle’of prism is A. Ifthe angle of minimum deviation is equal to the angle of the prism, then the value of Ais : (A) 30° (©) 60° Sob (By 45° (0) 75° 49. R.L. of 8 prism ee and the angle of prism is 60°. The limiting angle of incidence of a ray that will bbe tansmitted through the prism is (ayzor (8) 45° (C)15* (0) 50% ‘SO. The angle of a prism is 60° and the index of refraction of glass with air is 1.5. If the angle of incidence on the first face is 1, and the angle of emergence at the second face is I, then the Prism produces minimum deviation when Miso @Mh>h Omeh ©) WD Sot 51. In a thin prism of glass (refractive index 1.5) Which of the following relations between the angle of minimum deviation 8, and angle of refraction will be correct : Won =e (8) by = 1.56 (©) dy = 2 (0) m= 72 Sot ‘SECTION (E) : RAFRACTION BY ‘SPHERICAL SURFACE '52. The image for the converging beam after refraction through the curved surface is formed at : '53. A planoconcave lens is placed of 2 paper on which a flower is drawn: How far above its actual position does the flower appear to be ? nana olen = a2 [wef Paper (B)1S5¢m (C) SOem (0) none (A) 10m Sol. ‘SECTION (F): LENS AND COMBINATION OF LENSES/LENS & MIRRORS: 54.4 thin lens of focal length f and its aperture diameter d, forms a real image of intensity I. Now the central part of the aperture upto diameter (2) is blocked by an opaque paper. The focal length and image intensity would change to mis ed a won 5S. A thin symmetiical double convex /A\ lens of power P is cut into three part, ‘asshown inthe figure. Power ofA is = cz (8) ‘Sol. 5 "wp '56.A plano convex lens has a curved surface of ra- dius 100 cm. If p = 1.5, then the focal length of the lens is (A) 50cm (8) 100m (©) 200em (0)500.cm ‘Sol, 57. A lens of power + 2.0 Dis placed in contact with another lens of power - 1.0 D. The combination will behave like (A) a converging lens of focal length 100 cm (8) 2 diverging lens of focal length 100 cm (C) a converging lens of focal length 50 cm (0) a diverging lens of focal length 50 cm. Sol. 5B, A biconvex lens has 2 focal length of 10 cm. It is Cut in half and two pieces are placed as shown. The focal length of the final combination is : (ayi0 (8)20 (240 (D) Nota tens Sol. ‘58. Parallel beam of light is inéident on 2 system of two convex lenses of focal lengths f, = 20 cm and f, = 10 cm, What should be the distance between the two lenses so that rays after refraction from both the lenses pass undeviated 60. A pin Is placed 10 cm in front of convex lens of focal length 20 cm and refractive index 1.5. The sur- face of the lens farther away from the pin is silvered and has a radius of curvature of 22 cm. How far from the lens is the final image formed ? (A)iLcm — (8)12em —(C) 13. en Sol. (0) 14cm 61. When the object is at distances u, and u, the Images formed by the same lens are real and virtual respectively and of the same size. Then focal length of the lens.is ; Hee (& Fyre (©) fav, (0) 20y+6,) Sol 62. The height of the Image formed by a converging Jens on a screen is 8 cm. For the same position of the object and screen again an image of size 12.5 cm is formed on the screen by shifting the lens. The height of the object (A) 625/32 cm (8)64/12.5an (C) 10cm (0) none Sol, ‘SECTION (G) : DISPERSION OF LIGHT 63. The dispersion of light in a medium implies that: (A) lights of different wavelengths travels with different ‘speeds in the medium (8) lights of different frequencies travel with different ‘speeds in the medium {C) the refractive index of medium is different for different wavelengths {D) all of the above. Sol. 64, Critical angle of light passing from glass to air is minimum for (A)red (C) yellow ‘Sol. (8) green {D) violet ‘65. A plane glass slab is placed over various coloured letters. The letter which apears to be raised the least is: (A) violet (Oyred Sol, (8) yellow (0) green 66. A medium has n, = 1.56, 9, = 1.44, Then its dispersive power is. OZ ‘Sol. (8) ——(€)0.03—_(D) none 67. The refractive index of flint glass for blue line is 1.6333 and red line is 1,6161, then dispersive Power of the glass is : (A) 0.0276 (8) 0.276 (0) 2.76 (0) 0.106 Sob 68. Indicate the correct statement in the following (A) The dispersive power depends upon the angle of prism (B) The angular dispersion depends upon the angle of the prism (C) The angular dispersion does not depend upon the dispersive power (0) The dispersive. power in vacuum is one Sol. (68.-Which of the following diagrams shows correctly the dispersion of white light by 2 prism ? WY wd Pe of [REASONING TYPE] 70. Statement-I : If a source of light is placed in front of rough wall its image is not seen. ‘Statement-II : The wall does not reflect light. (A) Statement-1 is true, statement-2 is true; state- ‘ment-2 is correct explanantion for satemient-1 (B) Statement-1 is true, statement-2 is true; state ‘ment-2 is NOT correct explanantion for satement-1 (C) Statement - 1 is true, Statement - 2 is false. {D) Statement - 1 is false, statement - 2 is true. Sol. 71. Statement-I: As the distance x of a parallel ray from axis increases, focal length decreases ‘Statement-I : Asx increases, the distance from pole to the point of intersection of reflected ray with principal axis decreases (A) Statement-1 is true, statement-2 is true/"statew ‘ment-2 15 correct explanantion for satement-1 (B) Statement-1 is true, statement-2 is true; state- ‘ment-2 is NOT correct explBnantion for satement-1 (C) Statement=1 is true, Statement-2'ts false. (D) Statement-1 Is false, statement-2 Is true. Sol. 72. Statement-I : When an object dipped in a liquid is viewed normally, the distance between the image ‘and the object Is indepedent of the height of the liquid above the object. ‘Statement-II : The normal shift is independent of the location of the slab between the object and the observer. (A) Statement-1 is true, statement-2 is true; statement-2 is correct explanantion for satement-1 (B) Statement-1 is true, statement-2 is true; ‘statement-2 is NOT correct explanantion for satement-1. (C) Statement-1 Is true, Statement-2 is false. (0) Statement-1 is false, statement-2 (s true. Sol. 73. Statement-I : When two plane mirrors are kept perpendicular to each other as shown ( O si the point object), 3 image will be formed. ‘Statement-II: In case of multiple reflection, image of one surface can act as an object for the next surface. (A) Statement-1 is true, statement-2 is true; state- ment-2 is correct explanantion for satement-1 (8) Statement-1 Is true, statement-2 is true; state- ‘ment-2 's NOT correct explanantion for satement-1 (C) Statement-1 is true, Statement-2 is false. (D) Statement-t is false, statement-2 is true. Sol. 74, Statement-I : A piece of paper placed at the Position of a real image of a virtual object of intense light will burn after sufficient time. Statement-IT : A virtual object is that point where the incident rays appesr to converge and a real image is that point at which reflected/ refracted rays actu- ally converge. (A) Statement-1 is true, statement-2 Is true; state- ‘ment-2 is correct explaniantion for satement-1 (8) Statement-1 is true, statement-2 is true; state- ‘ment-2 is NOT correct explanantion for satement-1 (C) Statement-1 is true, statement-2 is false. (D) Statement: is false, Statement-2 Is true. Sob Exercise ou "SECTION (A): PLANE MIRROR. 4. When a plane mirror AB is placed horizontally on level ground at a distance of 60 metres from the foot Of 8 tower, the top of the tower and Its image in the mirror subtends, an angle of 90° at B. The height of the tower is errr (A)30 metre (8) 60 metre (C)90metre (0) 120 metre. Sol. 2. A unnumbered wall dock shows time 04 : 25 : 37, where Ist term represents hours, 2nd represents, minutes & the last term represents seconds, What time will its image in a plane mirror show. (A) 08 : 35: 23 (8) 07: 35:23 (©) 07: 34:23 (0) None of these Sol. JEE ADVANCED - OBJECTIVE ] '3. Two plane mirrors of length L are separated by distance Land a man M, Is standing at distance L from the connecting line of mirrors as shown in figure. ‘Aman M, 1s walking in a straight line at distance 2L parallel to mirrors at speed u, then man M, at © will be able to see image of M, for total time 4. A person Is standing in a room of width 200 cm. A plane mirror of vertical length 10 cm is fixed on a wall in front of the person. The person looks into the mirror from distance 50 em. How much width (height) of the wall behind him will he be able to see : (assume that fhe uses the full mirror) {D) None {A)30em — (B)40cm —(C)S0Em 5.1n the diagram shown, all the... , velocities are given with respect to earth. What is the relative velocity of the image in mirror (1) with respect to the image in the mirror (2)? The mirror (1) forms an angle p with the One vertical. Meer (A) 2vsin2p, (8) 2V sing (C)2v/sin2p (0) none Sol. 6. Two plane mirrors are inclined to each other at an angle 60°. If a ray of light incident an the first mirror isparallel to the second mirror, itis reflected from the second mirror. (A) Perpendicular to the first mirror (8) Parallel to the first mirror (C) Parallel to the second mirror {D) Perpendicular to the second mirror. 7. Two mirrors are inclined at an angle @ as shown in the figure. Light ray Is incident parallel to one of the mirrors. The ray will start retracing its path after third reflection if e 45° (8) = 30° (C)0= 60° (0) all three 8. There are two plane mirror with reflecting surface facing each other. Both the mirrors are moving with ‘speed v away from each other. A point object is placed between the mirrors. The velocity of the image from due to n-th reflection will be (yay (8) 2nv Sob, (C)3nv (0) dav ‘9. Two plane mirrors are placed parallel to each other ‘at a distance L apart. A point object O 1s placed ‘between them, at a distance L/3 from one mirror. Bath ‘mirrors form multiple images. The distance between any two images cannot be (ay3u2 (8) 20/3 Sol. coz (0) None 10. Images of an object placed between two plane mirrors whose reflecting surfaces make an angle of 90° with one another lie on a : (A) straight tine (8) 219-229 curve (Qarde (0) elipse Sol. 12, A person's eve is at a height of 1.5 m. He stands in front of a 0.3m tong plane mirror which is 0.8m above the ground. The length of the image he sees of himseltis : (A)1.Sm. (10m (C)0.8m — (0) 0.6m 12. Aman of height ‘h’is walking away from a street lamp with a constant speed ‘Vv. The height of the street lamp is 3h. The rate at which of the length of tthe man's shadow is increasing when heis at a distance 10h from the base of the street lamp is : (v2 (8) W/3 (C2 ows Sol. 13. A boy of height 1.5 m with his eye level at 1.4m stands before a plane mirrar of length 0,75.m fixed on the wall, The height of the lowerredge of the mirror above the floor is 0.8m. Then: (A) the boy will see his full image (B) the boy connot see his hair (C) the boy cannot see his feet (D) the boy cannot see neither his hair nor his feet. ‘Sol. 414, plane mirror is moving with veloaty 4i -§)« ‘A point object in front of the mirror moves with a velocity 3i+4]-5k- Here j is along the normal to the plane mirror and facing towards the object. The velocity of the image is. (A) -3)-4j 0 (©) 31-4) 19% ‘Sol. (8) 3i-ajoani (0) 71-9) 4% 15. A man of height 170 cm wants to see his complete image in a plane mirror (while standing). His eyes are {at a height of 160 cm from the ground. (A) Minimum length of the mirror = 80 cm (B) Minimum length of the mirror = 85 om (C) Bottom of the mirror should be at a height 80 cm orless (0) Bottom of the mirror should be at a height 85 cm Sol. 116. A flat mirror M is arranged parallel to a wall W at a 4, “1 (A) concave & placed towards right I (B) concave & placed towards left of I (C) convex and placed towards right of I (D) convex & placed towards left of 1 Sob 25. A point object at 15 cm from a concave mirror of radius of curvature 20 cm is made to oscillate along the principal axis with amplitude 2 mm. The amplitude ofits image will be (A2mm — (B)4mm_—(C)BmMm Sol. (0) none 226. A luminous point object is moving along the principal {axis of a concave mirror of focal length 12 em towards It When its distance from the mirror is 20 cm its velocity is 4 om/s. The velocity of the image in cm/s at that instant is (A) 6, towards the mirror (B) 6, away from the mirror (C) 9, away from the mirror (D) 9, towards the mirror. Sol. 27. A point object on the principal axis at a distance 15 cm in front of a concave mirror of radius of curvature 20 cm has velocity 2 mm/s perpendicular to the prinapal axis. The velocity of image at that instant will be : (A)2 mms. (8)4 mm/s (C)8 mms (D) none of these Sol. 28. The origin of x and y coordinates Is the pole of 2 concave mirror of focal length 20 cm. The x-axis Is the optical axis with x > 0 being the real side of mirror. A point object at the point (25 cm, 1 cm) is moving with 2 velocity 10 cm/s in positive x-direction. The velocity of the image in cm/s is approximately (A) 801-8) (8) 1601-8) (C) -1601-8) (0) 1601-4) Sol, 29. The circular boundary of the concave mirror subtends a cone of half angle 0 at its centre of ‘curvature. The minimum value of @ for which ray indigent ‘on this mirror parallel to the principle axis suffers reflection more than one is (ase (C) 60° (a) 30° (0) 75° ‘Sol, ‘30. An object! placed in front of a convex mirror at @ distance of 50 cm. A plane mirror is introduced covering the lower half of the convex mirror. If the distance between the object and the plane mirror is 30 cm, itis {ound that there is no gap between the images formed bby the two mirrors. The radius of the convex mirror is: (A)12.5cm (8)25cm —(C) Sem (0) 100em Sol, 31. In the figure shown find the total magnification after two successive reflections first on M, & then on ™, oan” 300n (0)- (B)-2 (+2 (ayen Sol, ‘32. In the figure shown ifthe object 0’ moves towards the plane mirror, then the image I,(which is formed after successive reflections from M, & M, respectively) ° My M, (A) towards right (8) towards teft (C) with zero velocity _—_(D)cannot be determined 33. A ray of light is incident on 8 concave mirror. It is ‘parallel to the principal axis and its height from principal axis Is equal to the focal length of the mirror. The ratio of the distance of point B to the distance of the focus from the centre of curvature is (AB Is the ‘reflected by) A & @ > 2 2 Os ‘Sob, 34. The image (of a real object) formed by 8 concave ‘mirror is twice the size of the abject. The focal length of the mirror is 20 cm, The distance of the object from the mirror is (are) (A)10cm (8) 30cm Sol, (C)25em (0) 15am 35. Inthe figure shown consider the first reflection at the plane mirror and second at the convex mirror. AB is object. velocity ‘*_8 c ‘toon “toon 20cm ‘0am (A) the second image's rea, inverted of 1/Sth magnification (B) the second Image is virtual and erect with magnification 1/5 (C) the second image moves towards the convex mirror, (0) the second image moves away from the convex rraroc Sol, 36. A reflecting surfaces represented by the equation yf 2 ae{ 28) 6 vB Sol. )nomne (0)u8= ud '39. Aray of light moving along the unit vector (~ | - 2j) undergoes refraction at an interface of two media, which ts the x-z plane. The refractive index for y > 0 is 2 while for y < 0, itis /§ 12. The unit vector along which the refracted ray moves is. esis) i (a SEP wy Aish (cy es) (D) None of these 40. How much water should be filled in a container of 21 cm in height, so that it apears half filed (of total height of the container) when viewed from the top of the container? (Assume near normal incidence and y, = 4/3) (A) 8.0 cm (8) 10.5em (C)12.0 em (0) 14.0 cm Sol. ‘41, Amark at the bottom of a beaker containing liquid appears to rise by 0.1m. The depth of the liquid is 1m. the refractive index of liquid is (Ay1.33 (8) 9/10 (C) 10/9 Sol. (ys 42. Aparallel sided block of glass of refractive Index 1.5 which is 36 mm thick rests on the floor of @ tank which is filled with water (refractive index = 4/3.) The difference between apparent depth of floor at A & B when seen from vertically above 1s equal to (A2mm — (B)3mm_—(C)4mm Sol (0) none 43. An under water swimmer isat.a depth of 12 m below the surface of water. A bird is at a height of 18 im from the surface.of water, directly above his eyes. For the swimmer, the bird appears to be at a distance X from the surface of water, (Refractive index of wa- ters 4/3.) The value of X is 2mm (B)12m_—(C)18M_— (DY 9m Sol, ‘44. A concave mirror Is placed on 3 horizontal table, with its axis directed vertically upwards. Let O be the pole of the mirror and C its centre of curvature. A point object is placed at C. Its has a real image, also located at C (a condition called auto-collimation). If the mirrors now filled with water, the image will be : (A) real, and will remain at C (B) real, and located at 2 point between C and = (C) virtual, and located at a point between C and O (D) real, and located at a point between C and O 45. A bird is flying 3 m above the surface of water. If the bird is diving vertically down with speed = 6 m/s, his apparent velocity as seen by a stationary fish underwater is (ABm/s (8) 6 m/s Sol. (C)12m/s (D)4 m/s 46, ven that, velocity of ghtn quart 5 x 10 ifs and velocy ooh nglycine =) 2 430 ys. Now 2 slabimiade of quartz isiplaced'if glycerine as shown. The shift of the object produced by slab is yeom Ciyeerinef | Srearne Observer Cbject 20am Quartz (A)6om — (8) 3.55em (C)9em (D)2em Sol. ‘47. Aight ray is incident on a transparent sphere of index = /2, at an angle of incidence = 45°. What is the deviation ofa tiny fraction of the ray, which enters the sphere, undergoes two internal reflections, and then refracts out into air? (A)270* —(B) 240" (C) 120" (0) 180" Sok 48. In the figure ABC is the cross, ‘section of aright angled prism and 8 BCDE is the cross: section”ofia glass slab. The value of 6 so that light incident normally on the face ‘AB does not cross the face BC is (given sin-i(3/5) = 37°) rs (A) 0< 379 (8) 8>37° (Q)o=s3° (D) 9< 53° Sol. 49. A cubical block of glass of refractive index n, is in contact with the surface of water of refractive index 1, A beam of light is incident on vertical face of the block (see figure). After refraction, 2 total internal reflection at the base and refraction at the opposite vertical face, the ray emerges out at an angle 8. The value of dis given by (A) swe yr (8) tan < yn a 1 1 Co) m0 Fy (Od ne TS Sol. 50. A vertical pencil of rays comes from bottom of a tank filled with a liquid, When itis accelerated with an acceleration of 7.5 m/s’, the ray \s seen to be totally reflected by liquid surface. What is minimum possible refractive index of liquid ? {(A) slightly greater than 4/3 (€) slighty greater than 5/3 (C) slightly greater than 1.5 (0) slightly greater than 1.75, Sol, ‘51. A ray of light is incident normally on one face of 30® ~ 60° ~ 90° prism of refractive index 5/3 immersed in water of refractive index 4/3 as shown in figure. (A) The exit angle u, of the ray Is sin“(5/8) (8) The exit angle 6, of theray Is.sin'( 5 443 ) (C) Total internalfeftection at point ceases if the tefractive|index offwater is increased to 5/2/3 by {dissolving some substance. (0) Total internal reflection at point P ceases if the tefractive Index of water Is Increased to 5/6 by ‘dissolving some substance. ‘Sol. 52. A ray of light in a liquid of refractive index 1.4, approaches the boundary surface between the liquid ‘and air at an angle of incidence whose sine is 0. Which of the following statements is correct about the behaviour of the light {A) Its impossible to predict the behavior of the light ray on the basis of the information supplied (8) The sine of the angle of refraction of the emergent ray will less than 0. {C) The ray will be internally reflected (0) The sine of the angle of refraction of the emergent ray will be greater than 0.8. '53. The figure shows ray incident at an angle i = 3/3. If the plot drawn shown the variation of |r ~1| versus | (C) u?~ ‘Sol, a/v =, (F= angle of refraction) wm 2 FF (8) the value of 0, = v6 3/3 (D) the value of k, is 1 (A) the value of k, (C) the value of 0, Sol. ‘54, Inthe diagram shown, a ray of light is incident on the interface between 1 and 2 at angle slightly greater than critical angle. The light surfers total intemal reflection at this interface. After that the light ray falls at the interface of 1 and 3, and again it suffers total internal reflection. Which of the following relations should hold true? n> men, (8) 3 (D) p2 +3 > ud we > wt who) ‘SECTION (D): REFRACTION BY PRISM 55. A ray of monochromatic light is incident on one refracting face of a prism of angle 75*. It passes through the prism and is incident on the other face at the critical angle. If the refractive index of the materia of the prism is \2, the angle of incidence on the first, face of the prism is (ay30"(B)45° (p60 =D) OF ‘56. Aprism having refractive index /3 and refracting angle 30°, has one of the refracting surface polished. ‘A beam of light incident on the other refracting surface will retrace its path if the angle of incidence ts (A) 0° (8) 30° qoy4se (0) 60" Sol. ‘57. Atriangular prism of glass is shown in figure. Aray Incident normally on one face is totally reflected. If 0 45°, the index of refraction of glass is (A) Less than 3 (8) Equal to Jz (C) Greater than JZ (0) None of the above. 8. A ray of light Is Incident normally on the first refracting face of the prism of refracting angle A. The ray of light comes out at grazing emergence. If one half of the prism (shaded position) is knocked off, the seme ray will (AY eherge at an angle of emergence sir ( Se0cA/2] (B) not emerge out of the prism (©) erage san ale tereraence sin (Seca 4) (0) None of these Sol 58. A ray of light is Incident at an angle 60° on the face of a prism having refractive angle 30*.The ray emerging out of the prism makes an angle 30° with the incident ray . 0 through which it emerges trom the surtece . (ayor (0) 60° (e)30°(cyase 60. A ray of light is incident normatly on one face of ‘an equilateral prism of refractive index 1.5. The angle of deviation is (A) 30° (c) 60" ‘Sol. (845° (0) 75° 61. Light ray is Ineident ofa prism 6f angle A= 60° and refractive index yw = /2 «The angle of incidence lat which the efiergent ray grazes the surface is given en (3S) wrsm(3) ‘62. The angle of a prism is A and one of Its refracting surfaces is silvered. Light rays falling at an angle of Incidence 2 A on the first surface return back through the same path after suffering reflection at the second (ilvered) surface. The refractive index of the mate- ‘al of the prism is (A)2sin A (8) 2cos A (©) 1/2 cos A (C) tana ‘Sol 1630A prism of Fefractive index /3 has 8 refracting ‘angle of 30*, One of the refracting surfaces of the ‘prism is polished. A beam of monochromatic light will retrace its path if its angle of incidence on the re- fracting surface Is (yor (@)30° = (c)45* (0) 60" ‘Sol. 64, The maximum refractive index of a material of 2 prism of apex angle 90° for which light will be transmitted is : O# WS @is (0) None ‘A prism having an apex angle of 4° and refractive index of 1.50 is located in front of a vetical plane mirror as shown. A horizontal ray of light is incident on the prism. The total angle through which the ray is deviated is Pp ™ (A) 4° dockwise (8) 178° dockwise (C) 2° clockwise (0) 8° clockwise 66. A thin prism of angle 5* is placed at a distance of 10 em from abject. What is the distance of the image from object? (Given y of prism = 1.5) (A) x/8cn (8)s/120 (C) 5x36 em (D) x/7 em Sol. 67. An equilateral prism deviates a ray through 40° for two angles of incidence differing by 20°. The possible angle of incidences are : (a) 40" (8) 50° (20° (0) 60" ‘Sol, 68. A prism of refractive index /3 has refracting angle 60°. In order that a ray sulfers minimum deviation it should be incident at an angle (ayase (6) 90 (30° (D) none Sol. 69. For refraction through a small angled prism, the angle of deviation : (A) increases with the increase in R.I. of prism. (8) will decrease with the increase in R.I. of prism. (C) is directly proportional to the angle of prism. (0) will be 20 for a ray of R.L. = 2.4 if itis D for aray OR. = 1.2 Sol. 70. For the refraction of light through a prism (A) For every angle of deviation there are two angles of incidence. (8) The light travelling inside an equilateral prism is necessarily parallel to the base when prism is set for ‘miniewum deviation. (C) There are two angles of incidence for maximum deviation. {D) Angle of minimum deviation will increase If refractive Index of prism is increased keeping the outside medium unchanged if >i Sol. ‘SECTION (E): REFRACTION BY SPHERICAL ‘SURFACE 74. Afish is near the centre ofa spherical water filed fish bowl, A child stands in air ata distance 2 R (Ris radius of curvature of the sphere) from the centre of the bow At whet distance from the centre would the child's nose appear to the fish situated at the centre (Ru otwater = 4) (Ayan (8) 2R Sol. ()3R (OR 72. Aconcave spherical surface of radius of curvature 10.cm separates two medium x & y of refractive index 4/3 & 3/2 respectively. If the object is placed along principal axis in medium X then x (A) image is always real (B) image is real if the object distance is greater than ‘90on (C) image is always virtual (D) image is virtual if the object distance is less than 900m Sol. 73. A spherical surface of radius of curvature 10 cm separates two media X and ¥ of refractive indices 3/2 and 4/3 respectively. Centre of the spherical surface lies in denser medium. An object is placed in medium X. For image to be real, the object distance must be (A) greater than 90cm’ (8) less than 90 cm (C) greater than 80cm —_(D) less than 80 cm Sol. 74. A beam of diameter ‘d’ is incident on a glass hemisphere as shown. If the radius of curvature of the hemisphere is very large in comparison to d, then the diameter of the beam at the base of the hemisphere mae . (Ces (A) 3/40 (ae (a3 (0) 2/34 a 75 A concave spherical refracting surface separates two media glass and air (u,., = 1.5). If the Image is to be real at what minimum distance u should the object be placed in glass ifR is the radius of curvature ? (Aue 3R (B)u> 2k (Chu<2R (o)u ny, then there cannot be a real image of real object. (8) if u, > ,, then there cannot be 2 real image of virtual object. (C) if, > 44, then there cannot be a virtual image of virtual object. (0) ify > 44, then there cannot be 2 real image of real object. Question No. 81 to 83 (3 questions) ‘Accurved surface of radius R separatestwo medium of ‘refractive indices 1, and », a$ shown in figures A and B RES) an 1 {§1. Choose the correct statement(s) related to the ‘real image formed by the object O placed at a distance x, a8 shown in figure A (A) Reat image is always formed irrespective of the Position of object if u,>u, (B) Real image is formed only when x > R (C) Real image is formed due to the convex nature of the interface irrespective of u, and u, (D) None of these ‘Sol, 182. Choose the correct statement(s) related to the virtual image formed by object O placed at a distance x, a8 shown in figure A (A) Virtual mage is formed for any position of Of, <1, (8) Virtual image can be formed if x > Rand n, 1, (D) None of these Sol. 3. Identity the correct statement(s) related to the: formation of images of a real object O placed at x from the pole of the concave surface, as shown. in figure B (A) If, > 4, thenvirtual image is formed forany value of (8) Ifu, > n. then virtua image is formed ifx < (C) Iu, < 4, then real image is formed for any value ofx (0) none of these Sol. ‘SECTION (F): LENS AND COMBINATION OF LENSES/LENS & MIRRORS. 184. Two symmetric double convex lenses A and 8 have ‘same focal length, but the radii of curvature differ so that R, = 0.9R,. If, = 1.63, find ny. (Ay? (eyé (as Sok. (04/3 BS. When a lens of power P (in air) made of material of refractive index wis immersed in liquid of refractive Index y,. Then the power of lens is : (nt (DY hone of these 186. An object Is placed at 10 cm from a lens and real image Is formed with magnification of 0.5. Then the lens is : (A) concave with focal lenth of 10/3 cm (8) convex with focal length of 10/3 cm (C) concave with focal length of 10 cm (0) convex with focal length of 10 cm Sol. 87. A thin linear object of size 1 mm is kept along the Principal axis of a convex lens of focal length 10 cm. ‘The object is at 15 cm from the lens. The length of the image is : (A)imm Sol. (8)4mm_=—(C)2mm_— (0) Bm ‘Question No. 88 to 90 (3 question) A turnip sits before a thin converging lens, outside the focal point of the lens. The lens is filled with a transparent gel so that it is exible; by squeezing its ends toward its center (as indicated in figure (a), you can change the curvature of its front and rear sides. Figura) 8B. When you squeeze the lens, the image (A) moves towards the lens (8) moves away from the lens (C) shifts up (D) remains as itis Sol. 189. The lateral height of image (A) increases (8) decreases (C)remains same —_(D) data insufficient ‘Sol, 90. Suppose that the image must be formed on a card which is at a certain distance behind the lens {figure (B)], while you move the turnip away from the lens, then you should on igre) (A) decrease the squeeze of the lens. (B) increase the squeeze of the lens (C) keep the card and lens as itis (D) move the card away from the lens Sob, 91. If an object is placed at A(OA > f); Where fis the focal length of the lens the image is found to be formed at B. A perpendicular Is erected at O and C Is chosen (on it such that the angle /BCA is a right angle. Then the value of f will be & 4] / SS (ay as/oc* (©) (OCKABY/AC#BC (B) (AC)(BC OC (0) 0c:/AB 92. A converging lens of focal length 20 cm and diameter S.cm is cut along the line AB. The part of the lens shown shaded in the diagram is now used to form {an image of a point P placed 30 cm away from it on the line XY. Which is perpendicular to the plane of the ens. The image of P will be formed. fae (A) 0.5 cm above XY (C) on XY Sol. (8) cmbelowxy (©) 1.5 am below x¥ 93. A point object Is kept at the first focus of a convex lens. If the tens starts moving towards right with a constant velocity, the image will object F (A) always move towards right (8) always move towards left (C) first move towards right & then towards left. (D) first move towards left & then towards right. 94 two loo-cones ese ach fa ent 30 cm retactve index 3 ae laced a shown nthe 4 space lett, water (R!-$) is flea! The whole ‘arrangement isin alr. The optical power of the system is (in diopters) : (A)6.67 (8) -6.67 (€)33.3. (0) 20 Sol, 9S. An object is placed at a distance of 10 em from a co-axial combination of two lenses Aand B in contact. The combination forms a real image three times the size of the object. If lens B Is concave with a focal length of 30 cm, what is the nature and focal length of lens A? (A) Convex, 12 em {(C) Convex, 6 em Sol. (8) Concave, 12 em (D) Convex, 18 cm 96. The curvature radii of a concavo-convex glass lens are 20 cm and 60 cm. The convex surface of the lens is silvered. With the lens horizontal, the concave surface is filled with water. The focal length of the effective mirror is (sof glass = 1.5, » of water = 4/3) (A) 90/13 cm (8) 80/13. on’ (C) 20/3 cm (0) 45/8 em ‘Sol, 197. An object is placed at a distance of 15 cm from a ‘convex lens of focal length 10 cm. On the other side of the lens, a convex mirror is placed at its focus such that the image formed by the combination coincides. with the object itself. The focal length of the convex mirrors A <(é v7 (A)20cm (8) 10cm (C)1Sem_— (0) 30am '98. An object is placed in front of a thin convex lens of focal length 30 cm and a plane mirror is placed 15 ccm behind the lens. If the final image of the object coincides with the object, the distance of the object from the lens is (A)60cm —(8)30cm Sol, (C)aSem | (0)25an 98. Look at the ray diagram shown, what will be the {focal length of the 1* and the 2~ lens, if the incident light ray passes without any deviation? (A) Sem and -10em (C)-Sem and +Sam () +Scmand +10em (0) +Scmand +Sem 100. An object is placed in front of a symmetrical convex lens with refractive index 1.5 and radius of curvature 40 cm, The surface of the lens further away from the object Is silvered. Under auto-collimation condition, the object distance is (A)20cm = (B)10em_—(C)40em_—(D) Sem Sol. 101. A planoconvex lens, when silvered at its plane ‘surface is equivalent to a concave mirror of focal length 28cm. When its curved surface Is silvered and the plane surface not silverted, it is equivalent to a concave mirror of focal length 10cm, then the refractive index of the material of the lens is : ayers (B)14/9 (C)17/9 Sol. 102. In the above question the radius of curvature of the curved surface of plano-convex lens is : 4 on ©) Bon 3 200 «Fon (0) Sem Sot. 103. A screen is placed 90 cm from an object. The image of an object on the screen is formed by a convex lens at two different locations separated by 20 cm. The focal length of the lens is (A) 18 cm (B)2h4 cm (c) 60cm (0) 85.6.cm Sol. 104. In the above problem, if the size of the image formed at the positions are 6 cm and 3 cm, then the highest of the object is (A) 42cm (8)4.5am (C)Som (D) none of these Sol. 105. Which of the following cannot form real image of 2 real object ? (A) concave mirror (C) plane mirror (8) convex mirror (0) diverging lens 206. The radius of curvature of the left & right surface of the concave lens are 10 cm & 15 cm respectively. The radius of curvature of the mirror is 15 cm. ‘l= wreaay (A) equivalent focal length of the combunation is =18 cm (B) equivalent focal length of the combination is +36 cm (C) the system behaves like a concave mirror (0) the system behaves like a convex mirror. ‘Sol. 107. If a symmetrical biconcave thin'tens is cut into ‘wo identical halves. They are placed indifferent ways, as shown : “TIO (A) three images will be formed in case (i) (8) two images will be formed in the case (i) (C) the ratio of focal lengths in (il) & (ii) is 1 (D) the ratio of focal lengths (im) (11) & (1H!) ts 2 Sol. 108. A convex lens forms an image of an object on screen. The height of the image is 9 cm. The lens is now displaced until an image is again obtained on the screen. The height of this image is 4 cm. The distance between the object and the screen is 90 cm. (A) The distance between the two positions of the lens is 30 em, (B) The distance of the object from the lens ists first position is 36cm. (C) The height of the object is 6cm. (D) The focal length of the lens is 21.6 cm. Sol. 108. A thin lens.with focal féngth to be used as a magnifying glass, Which of the following statements regarding the situations is true? (A) A converging fens may be used, and the object be placed at a distance greater than 2f from the lens. (8) A diverging lens may be used, and the object be placed between f and 21 from the fens. (C)A converging lens may be used, and the object be placed at a distance less than f from the lens. (0) A diverging lens may be used, and the object be placed at any point other than the focal point. Sob 1110. Which ofthe following can form diminished, virtual and erect image of your face. (A) Converging mirror (8) Diverging mirror (C) Converging lens. (0) Diverging lens Sol. 111, Which of the following quantities related to 2 Tens depend on the wavelength of the incident light? (A) Refractive index (8) Focal length (6) Power (D) Radil of curvature 142, A man wishing to get a picture of a Zebra photographed a white donkey after fitting a glass with black streaks onto the objective of his camera. {A) the image will look Ike a white donkey on the photograph. (8) the image will look like a Zebra on the photograph {C) the image will be more intense compared to the ‘case in which no such glass ts used. (0) the image will be less intense compared to the ‘case in which no such glass is used. Sol. SECTION (6): DISPERSION OF LIGHT 113. A thin prism P, with angle 4® made of glass of refractive index 1.54 is combined with another thin prism P, made of glass of refractive index 1.72 to produce’ dispersion without deviation. The angle of the prism P, is ase (cae (0)5.33° Sol. (8)2.6° 1114, Light of wavelength 4000 A is incident at small ‘angle on a prism of apex angle 4°, The prism has n, = 1.5 &n, = 1.48. The angle of dispersion produced by the prism in this light is ()0.192° (ay0.2° — (B)0.08° (0) none ‘Sol. 115. Two lenses in contact made of materials with dispersive powers In the ratio 2 : 1, behaves as an achromatic lens of focal length 10 em. The individual focal lengths of the lenses are (A) 5 cm, - 10 em (8)- Sem, 10cm (630m, -20¢m (0) 20cm, 10 em 146, A beam of light consisting of red, green and blue ‘and is incident on a right angled prism. The refractive index of the material of the prism for the above red, green and blue wavelengths are 1.39, 1.44 and 1.47 respectively. The prism will (A) separate part of the red colors from the green and blue colors. (6) separate part of the blue colors from red and green colors. {C) separate all the three colors from the other two colors. (0) not separate even partially any colour from the other two colors. 117. Its desired to make an achromatic combination of two lenses (L, & L,) made of materials having dispersive powers w, and a, ( '34, A beam of parallel rays of width b propagates in 138 at an angle 0 to its plane face. The beam width ‘after it goes over to air through this face is. If the refractive index of glass is u. lass ‘Air 35-A cubical tank (of edge /) and position of an observer are shown in the figure. When the tank is empty, edge of the bottom surface of the tank is Just Visible, An insect is at the centre C of its bottom surface. To what height 2 transparent liquid of refractive index y= /572 must be poured inthe tank so that the insect will become visible ? uA oe 36. Light from a luminous point on the lower face of @ 2 cm thick glass slab, strikes the upper face and the totally reflected rays outline a circle of radius 3.2 cm ‘on the lower face. What is the refractive index of the glass. ‘SECTION (D) : REFRACTION BY PRISM 37. A prism (n = 2) of apex angle 90° is placed in air (n= 1). What should be the angle cf incidence so that light ray strikes the second surface at an angle of 60° Sol. ‘38. Ref, index of a prism (A = 60°) placed in air (n = 1) isn = 1.5. Light ray is incident on this prism at an ‘angle of 60°. Find the angle of deviation. State whether {this is a minimum deviation. Gen: ain” F-35%, sir 0.4 = 25°, sin 0.6 = 372 ‘39. The cross section of o glass prism has the form of ‘an equilateral triangle. A ray is incident onto one of the faces perpendicular to it. Find the angle 0 between the Incident ray and the ray that leaves the prism. The refractive index of glass is = 1.5. Sou 40. The angle of refraction of a prism is 60°. A light ray emerges from the prism at the same angle as itis Incident on it. The refractive index of the prism is 1.5. Determine the angle by which the ray is deflected from its initial direction as a result of its passage through the prism. Sol 41, Find the angle of deviation suffered by the light ray shown in figure for following two condition The refractive index for the prism material is » = 3/2 (1) When the prism is placed in atr (y= 1) (Gi) When the prism is placed in water (1 = 4/3) Sol 42. A prism of refractive index 2 has a refracting langle of 30°. One of the refracting surface of the ‘prism is polished. For the beam of monochromatic light to retrace its path, find the angle of incidence on the refracting surface. Sol. ‘43, An equilateral prism deviates a ray through 23° for two angles of incidence differing by 23°. Find » of the prism 7 Sol. ‘44, A equilateral prism provides the least deflection angle 46° in air. Find the refracting index of an unknown liquid in which same prism gives least deflection angle of 30°, Sol ‘48. A ight angle prism (459 - 900° 450) of refractive index n has a plate of refractive Index n,(n, < A) cemented to its diagonal face. The assembly Is In air 2a ray is incident on, AB (see the figure). (i) Caleulate the angle of incidence at AB for which the Fay strikes the diagonal face at the critical angle. (ii) Assuming n = 1.352. Calculate the angle of Incidence at AB for which the refracted ray passes through the diagonal face undeviated. y 46. A prism of refractive index n, & another prism of refractive index n, are stuck together without a gap as shown in the figure. The angles of the prisms are as shown n, 8 n, depend on 2, the wavelength of light 108-10 18010" 7 according ton = 120+ py = 4S + where i isin nm. (1) Calculate the wavelength 9, for which rays incident at any angle on the interface BC pass through without bending at that interface. (i) For light of wavelength 2,, find the angle of Incidence | on the face AC such that the deviation roduced by the combination of prisms in minimum. Sol, SECTION (E): REFRACTION BY SPHERICAL ‘SURFACE 47. A spherical surface of radius 30 cm separates two transparent media A and B with refractive indices 4/3 ‘and 3/2 respectively. The medium A is on the convex side of the surface. Where should a point object be placed in medium A so that the paraxial rays becomes parallel after refraction at the surface? Sol. ‘48, Anarrow parallel beam of light is incident paraxially on a solid transparent sphere of radius r. What should be the refractive index if the beam is to be focused (2) At the surface of the sphere, (B) at the centre of the sphere. ‘Sot. 49. An extended object of size 2 cm is placed at a distance of 10 cm in air (n = 1) from pole, on the principal axis of a spherical curved surface. The medium on the other side of refracting surface has refractive index n = 2. Find the position, nature and size of image formed after single refraction through the curved surface. Toon SO. An object Is placed 10 cm away from a glass piece (n = 1.5) of length 20 cm bound by spherical surfaces of radii of curvature 10 cm. Find the position of final image formed after twice refractions. 51. There is a small air bubble inside a glass sphere (u = 1.5) of radius 5 om. The bubble is 7.5 cm below the surface of the glass. The spheres placed inside water tu $) suerte the tp srace tas £0 em below the surfaceiof water, The bubble is viewed normally from alr. Find the apparent depth on the bubble. '52. A narrow parallel beam of light is incident on a transparent sphere of refractive index'n’. If the beam finally gets focussed at a point situated at a distance = 2 x (radius of sphere) from the centre of the sphere, then find n? Sol. '53. A.uniform, horizontal beam of light is incident upon 2 quarter cylinder of radius R = 5 cm, and has a refractive index 21 J. A patch on the table for a distance'x’ from the cylinder is unilluminated, find the value of x’? uit ‘SECTION (F) : LENS AND COMBINATION OF LENSES /LENS & MIRRORS. ‘54, A double convex lens has focal length SO cm. The radius of curvature of one of the surfaces is double of the other. Find the radit, if the refractive index of the material of the lens is 2. Sol. 5S. Lenses are constructed by a material of refractive index 1.50. The magnitude of the radii of curvature are 20 cm and 30 cm. Find the focal lengths of the possible lenses with the above specifications. Sol. ‘56. Given an optical axis MN & the positions of a real ‘object A B and its image AB, determine diagramatically the position of the lens (its optical centre 0) and its fod. Is ita converging or diverging lens? Is the image real or virtual? 57 Athin lens made of a material of refractive index i, has a medium of refractive index », on one side and 2’ medium of refractive index y, on the other side. The lens is biconvex and the twa radit of curvature has equal magnitude R. A beam of light travelling parallel to the principal axis is incident on the lens. Where wil the Image be formed if the bearmis incident from (a) the medium », and (B) from the medium,, ? Sot SB. An object of height 6 cm is set at right angles to the optical axis of a double convex lens of optical power 5 d& 25 cm away fram the lens. Determine the focal length of the lens, the position of the image, the linear magnification of the lens, and the height of the image formed by it. '59.A5.0 diopter lens forms a virtual image which is 4 times the object placed perpendicularty on the principal ‘axis of the lens. Find the distance of the object from the lens. ‘Sol. (60. Aconverging lens and diversing mirror are placed at a Separation of 15 cm. The focal length of the lens. s 25 cm and that of the mirror is 40 om. Where should 2 point source be placed between the lens and the ‘mirror so that the light, after getting reflected by the ‘mirror and then getting transmitted by the lens, comes ‘out parallel to the principal axis ? Sol. 61. A converging lens of focal length 15 cm and 8 ‘converging mirror of length 10 cm are placed 50 cm apart. if a object of length 2.0 cm is placed 30 cm {from the lens farther away from the mirror, where will the final image form and what will be the size of the final image ? Sol (62. 2 identical thin converging’ lenses brought in contact so that their axes coincide are placed 12.5 ‘cm from an object: What is the optical power of the ‘system & each lensyif the real image formed by the ‘system of lenses is four times as large as the object? ‘Sol, 63. A point object Is placed at 2 distance of 15 cm from a convex lens. The Image is formed on the other side at a distance of 30 cm from the lens. When 2 ‘concave lens is placed in contact with the convex lens, the image shifts away further by 30 cm. Calculate the focal lengths of the two lenses. 64. A convex & a concave lens are brought It dose contact along thelr optical axes. The focal length of the convex lens is 10 cm, When the system is placed ‘at 40 cm from an object, a sharp image of the object {stormed on a screen on the other side of the system. Determine the optical power of the concave lens if the distance / between the object & the screen is, 1.6m, Sol. 65. A point object Is placed at a distance of 25 cm from a convex lens of focal length 20 cm. If a glass slab of thickness t and refractive index 1.5 s inserted between the lens and object. The image is formed at infinity. Find the thickness t ? Sol. 66. An object |s kept at a distance of 16 cm from the thin lens and the image formed is real. If the object is. kept at a distance of 6 cm from the same lens the Image formed is virtual. If the size of the image formed are equal, then find the focal length of the lens ? Sol. 67. Athin convex tens forms a real image of a certain object’p’ times its size. The size of real image becomes “q’Umes that of object when the lens is moved nearer to the object by a distance 'a'find focal length of the lens ? 68. In the figure shown, the focal length of the two thin convex lensesiis the same = f. They are separated by a horizontal distance 3f and their optical axes are displaced by a vertical separation 'é' (d << f), a5 shown. Taking the origin of coordinates O atthe centre of the first lens, find the x and y coordinates of the point where a parallel beam of rays coming from the left finally get focussed 7 Wu 69. A point source of light is kept at'a distance of 15 em from a converging lens, on its optical axis. The focal length of the lens is 10 cm and its diameter is 3 om. A screen is placed on the other side of the lens, Perpendicular to the axis of lens, at 2 distance 20 cm from it. Then find the area of the illuminated part of the screen ? Sol. 70. A glass hemisphere of refractive index 4/3 and of radius 4 cm is placed on a plane mirror. A point object {is placed at distance ‘d' an axis of this sphere as ‘shown. If the final image be at infinity, find the value of 'd. 71,,A double convex lens has focal length 25.0 cm in ‘air. The radius of one of the surfaces is double of the {BP Find thera of curvature tthe refractive index ‘of the material of the lens is 1.5. ‘Sol, 72, A plano convex lens (= 1.5) has a maximum thekness of 1 mm. If lameter ofits aperture is4 cm. (1) Radius of curvature of curved surface (Gi) its focal length in air 73. A plano-convex lens, when silvered on the plane side, behaves like a concave mirror of focal length 30 ‘cm. When itis silvered on the convex side, it behaves, like @ concave mirror of focal length 10 cm. Find the refractive index of the material of the lens, ‘Sol, ‘SECTION (G) : DISPERSION OF LIGHT ‘74. A certain material has refractive indices 1.56, 1.60 ‘and 1.68 for red, yellow and violet light respectively. (2) Calculate the dispersive power. (8) Find the angular dispersion produced by a thin prism of angle 6° made of this material. 75. A flint glass prism and a crown glass prism are to bbe combined in such 2 way that the deviation of the ‘mean ray is zero. The refractive index of flint and ‘crown glasses for the mean ray are 1.620 and 1.518 ‘respectively. Ifthe refracting angle of the flint prism is 6.08, what whould be the refracting angle of crown prism? ‘Sol, ‘SECTION (8): SPHERICAL MIRROR 41. An observer whose least distance of distinct vision Is°6,, views his own face in a convex mirror of radius of curvature 'r’. Prove that magnification produced 2. Athief is running away in a car with velocity of 20 m/s. A police jeep is following him, which is sighted by thief in his rear view mirror which is a convex mirror of focal length 10m. He observes that the image of jeep ismoving towards him with a velocity of Lom/s. If the ‘magnification of the mirror for the jeep at that time is 1/10. Find (2) actual speed of jeep (b) rate at which magnification is changing Assume that police jeep Is on axis of the mirror. can not exceed 3. Aluminous point P is inside a circle. A ray enters from P and after two reflections by the circle, return to P. If 8 be the angle of incidence, a the distance of P from the centre of the circie and b the distance of the centre from the point where the ray in its course crosses the diameter through P, prove that ‘4. An object is keption the principal axisiof a convex mirror of focal length 10 cm at 2 distance of 10 cm from the pole. The object starts moving at a velocity 20 mm/sec towrads the mirror at angle 30° with the Drincipal axis. What will be the speed of its image and direction with the principal axis at that instant ? 55. In the figure shown Lis a converging lens of focal length 10 em and M is a concave mirror of radius of ‘curvature 20 cm. A point object O is placed in front of the lens at a distance 15 cm. AB and CD are optical axes of the lens and mirror respectively. Find the distance of the final image formed by this system from the optical centre of the lens. The distance between CDBABIs lem. LEVEL - ‘SECTION (C) : REFRACTION IN GENERAL, REFRACTION AT PLANE SURFACE ANDT.LR. 6. Asurveyor on one bank of canal observed the image of the 4 inch and 17 ft marks on a vertical staff, which is partially immersed in the water and held against the bank directly opposite to him, coincides. Ifthe 17 femark and the surveyor's eye are both 6f above the water level, estimate the width of the canal, assuming that the refractive index of the water is 4/3. 7. Adircular disc of diameter d lies horizontally inside a metallic hemispherical bowl radius a. The disc is just visible to an eye looking over the edge. The bow! is ow filled with a liquid of refractive index y. Now, the whole of the disc is just visible to the eye in the same position. Show that d= 2 18: ray of ight ervelting in alr Is incident at grazing ‘Brigle (Incidence angle = 90°) on a medium whose refractive index depends on the depth of the medium. ‘The trajectory of the light in the medium is a parabola, Y= 2x°, Find, at a depth of 1 m In the medium, (i) the refractive index of the medium and (ii) angle of incidence 4 Two thin similar watch glass pieces are joined together, front to front, with rear portion silvered and the combination of glass pieces is placed at a distance 2 = 60 cm from a screen. A small object is placed normal to the optical axis of the combination such that its two mes magnified image is formed on the screen. If air between the glass pieces is replaced by water (y = 4/3), calculate the distance through which the object must be displaced so that a sharp image is again formed on the screen. 10. A thin plano-convex lens fits exactly into a plano ‘concave lens with their plane surface parallel to each other as shown in the figure. The radius of curvature of the curved surface R= 30 cm. The lens are made of difference material having refractive index y, = 3/2 and u, = 5/4 8s shown in figure. 30| “se wy wee (() ifplane surface of the plano-convex lens is silvered, then calculate the equivalent focal length of this system and also calculate the nature of this equivalent mirror (ii) An object having transverse length S cm in placed (on the axis of equivalent mirror (in part 1), ata distance 15 cm from the equivalent mirror along principal axis. Find the transverse magnification produced by equivalent mirror. 12. A,ray of light travelling in airs incident at grazing angle (incident angle = 90°) on a long rectangular slab of a transparent medium of thickness t= 1.0 (see figure). The point of incidence isthe ongin A (0, 0). The medium has a variable index of refraction n(y} siven by : ny) = (ky'? + 1)"?, where k = 1.0m”, The refractive index of air is 1.0. (i) Obtain a relation between the slope of the trajectory af the ray at @ point B(x, y) in the medium and the incident angle at that point. (it) Obtain an equation for the trajectory ¥(x) of the ray in the medium. (iii) Determine the coordinates (x,, y,) of the point P, ‘where the ray the ray intersects the upper surface of the slab-air boundary. (iv) Indicate the path of the ray subsequently. (Dee 95) ‘SECTION (D) : REFRACTION BY PRISM 12. An isosceles triangular glass prism stands with its bbase in water as shown. The angles that its two equal ‘sides make with the base are # each. An incident ray of light parallel to the water surface internally reflects at the glass-water interface and subsequently re- ‘emerges into the alr. Taking the refractive Indices of glass and water to be 3/2 and 4/3 respectively, show that 0 must be at least tan # F 25.99, 13. A parallel beam of light falls normally on the first face of a prism of small angle. At the second face it is partly transmitted and partly reflected, the reflected beam striking at the first face again, and emerging from it in a direction making an angle 6°30’ with the reversed direction of the Incident beam. The refracted ‘beam is found to have undergone a deviation of 1°15" from the original direction. Find the refractive index of the glass and the angle of the prism. 14, The refractive indices of the crown glass for violet and red lights are 1.51 and 1.49 respectively and those of the flint glass are 1.77 and 1.73 respectively. A prism of angle 6° ts made of crown glass. A beam of ‘white light is incident at a small angle on this prism, The other thin flint glass prism is combined with a ‘crown glass prism such that the net mean deviation is 1,5° anticlockwise. rown glass Tint glass Tm (j) Determine the angle of the flint glass prism. (ii) A screen is placed normal to the emerging beam at a distance of 2m from the prism combination. Find the distance between red and violet spot on the ‘screen. Which Is the topmost colour on screen. 415. A concave mirror has the form of a hemisphere with a radius of R= 60 cm. A thin layer of an unknown transparent liquid is poured into the mirror. The mirror liquid system forms one real image and another reat image is formed by mirror alone, with the source in a ‘certain position. One of them coinddes with the source and the other is at a distance of / = 30 cm from source. Find the possible value(s) refractive index of the liquid. ‘SECTION (F) : LENS AND COMBINATION OF LENSES/LENS @ MIRRORS 16. In the figure shown, find the relative speed of approach/separation of the two final images formed after the light rays pass through the lens, at the ‘moment when u = 30cm. The speed abject = 4 cm/s, The two lens halves are placed symmetrically watt. the moving abject. 40m oe 17. In the figufe shown “O' is point objéct. AB is Principal axis of the converging lens of focal length F. Find the distance of the final image'from the lens. 1418. The rectangular box shown is the place of lens. By looking at the ray diagram, answer the following ‘questions. (1) IFXis Sam then what isthe focal length of the tens ? (ii) Ifthe point O is 1 cm above the axis then what is the position of the image ? Consider the optical center of the lens to be the origin, 19, Two identical convex lenses L, and L, are placed at a distance of 20 em from each other on the common principal axis. The focal length of each lens is 15 cm and the lens L, isto the right of lens L,. A point object is placed at 2 distance of 20 cm on the left of lens L,, fon the common axis of two lenses. Find, where 9 convex mirror of radius of curvature 5 am should be placed so that the final image coincides with the object ? 20. A thin plano-convex. Lens of focal length Fis split into two halves, one of the halvesis shifted along the ‘optical axis. The separation between object and image planes is 1.8 m. The magnification ofthe image formed by one of the half lenses is 2, Find the focal length of the lens and separation between the two halves. Draw theray diagram forimage formation. [JEE 96] 21, Athin equiconvex lens of glass of refractive index = 3/2 & of focal length 0.3 m in air is sealed into an ‘opening at one end of a tank filled with water (j= 4/ 3). On the opposite side of the lens, a mirror is placed inside the tank on the tank wall perpendicular to the {ens axis, as shown in figure The separation between the lens and the mirror is 0.8 m. A small object is placed outside the tank in front of the lens at a distance of 0.9m from the lens along its axis. Find the position (relative to the lens) of the image of the object formed by the system, (097) PREVIOUS YEAR QUESTIONS 41. An object 2.4 m in front of a lens forms a sharp image on a film 12 cm behind the lens. A glass plate 1 ccm thick, of refractive index 1.50 is interposed between lens and film with its plane faces parallel to film. At what distance (from lens) should to be in sharp focus of film? (ATEEE 2012) (A)7.2m — (8)24m_ —(C)3Zm_— (0) 5.6m Sol. 2.A beaker contains water up to a height h, and kerosene of height h, above water so that the total height of (water + kerosene) Is (h, +h,). Refractive index of water sj, and that of kerosene is j1,. THe apparent shift in the position of the bottom of the beaker when viewed from above is (ATEE€ 2081) wol-Bpaat © (dye) Sol, 2, When monochromatic red light is used instead of blue light in a convex lens, its focal length will (ATEBE 2011) (A) does not depend on colour of light (B)increase (C) decrease (D) remain same Sol 4, Statement I : On viewing the dear blue portion of the sky through a Calcite Crystal, the intensity of transmitted light varies asthe crystal 1s rotated. ‘Statement II : The light coming from the sky is po- lortzed due t© scattering of sun light by particles in the atmospheré. The scattering s largest for blue tight. (AIEEE 2011) (A) Statement 1 is false, Statement Il is true (8) Statement I is true, Statement II is true (C) Statement I 1s true, Statement II is true, State- ‘ment IT is the correct explanation of Statement I (0) Statement I is true, Statement II is true; State- ‘ment Il is not correct explanation os Statement I ‘Sol. Let the x-2 plane be the boundary between two transparent media. Medium 1 in z > 0 has arefractive index of J3 and medium 2 with z < Ones. refractive Index of 3. A ray of ight in medium 1 given by the vector X = 6/33 + 8V3}-10K ts ncident on the plane of separation. The angle of refraction in medium 2 is (AIEEE 2011) (75% (0) 30° (ayase (8) 60" 6. A car is fitted with a convex side-view mirror of focal length 20 cm. A second car 2.8 m behind the first car is overtaking the first car at a relative speed of 15m/s. The speed of the image of the second car as seen in the mirror of the first one is (ATEEE 2011) t (ay gms (©) 10 m/s 1 (Qsms ()75™'5 Sol. 7. As the beam enters the medium, it will (AIEEE 2010) (A) diverge (B) converge {C) diverge near the axis and converge near the periphery (0) travel as a cylindrical beam Sol. 8. The speed of light in the medium is (AIEEE 2010) (A) minimum on the axis of the beam (B) the same everywhere in the beam {C) directly proportional to the intensity {(D) maximum on the axis of the beam 9. In an optics experiments, with the position of the object fixed, a student varies the positions of a con- vvex lens and for each position, the screen is adjusted to get a clear image of the object. A graph between the object distance u anc the image distance v, from the lens, is plotted using the same scale for the two ‘axes. A straight line passing through the origin and making an angle of 45° with the x-axis meets the ‘experimental curve at P, The coordinates of P will be (AIBEE 2009) f wenn @lE4) Oa. f¥ (Dy 4h4/) ‘Sol 10. A transparent solid cylinder rod has a refractive 2 Index of 5 Tt is surrounded by air. A ight ray is Incident at the mid point of one end of the rod as shown in the figure. A) ‘The incident angle 0 for which the light ray grazes ‘along the wall of the rod is (AIEEE 2009) wr) om tS) om(S) w(t) 11, A student measures the focal length of a convex lens by putting an object pin at a distance u from the lens and measuring the distance v of the image pin. The graph between u and v plotted by the student ‘should look like (AIEEE 2008) van) vam) “ J @) Ol ue) oF ule) v(am) v(am) «© (0) \ OF ufem) ‘ol uem), 12. Two lenses of power -15D and +50 are in Con- tact with each other. The focal length of the combi- nation is (AIEEE 2007) (a)-20cm (8)-10cn (€)+20am (0) +10 Sol. 13, The refractive index of glass is 1.520 for red light and 1.525 for blue light. Let D, and D, be angles of minimum deviation for red and blve light respec- tively in a prism of this glass. Then (AIEEE 2006) (A)D, <0, (8)D,=0, (CD, can be less than or greater than D, depending upon the angle of prism: (©) 0,>d, Sol 114, Afish looking up through the water sees the out- side world, contained in a circular horizon. If the re~ fractive index of water is 4/3 and the fish is 12 cm below the water surface, the radius of this circle in emis (AIEEE 2005) 36 (A) 3607) (BF CC) 36V5— (0) VS 18. Two point white dots are 1 mm apart on a black paper. They are viewed by eye of pupil diameter 3 mm. Approximately, what is the maximum distance at which these dots can be resolved by the eye ? [Take wavelength of light = 500 nm] (AIEEE 2005) Asm (Bim = (C)6m_— (0) 3m ‘Sol 126, Athin glass (refractive index 1.5) lens has optical power of ~5 D in air. Its optical power in a liquid me~ dium with refractive index 1.6 will be (AIEEE 2005) (1D (B)-1D_—(C)2SD__—_()-25D 17, _Alight ray is incident perpendicular to one face of 2.90° prism and is totally internally reflected at the glass-air interface. If the angle of reflection is 45°, we conclude that for the refractive index n as (ATEEE 2005) ney ®) n> V3 ' Ore () n< V2 Sol. 18. A plano-convex lens of refractive index 1.5 and radius of curvature 30 cm is silvered at the curved surface.Now, this lens has been used to form the im= ‘age of an object. At what distance from this lens, an ‘object be placed in order to have 8 real image the size of the object ? (AIEEE 2004) (A)200n —(8)30.cm (C) 60cm (0) 80cm Sol. 19. The angle of incidence at which reflected light is totally polarized for reflection trom air to glass (re- fractive index n), is (AIEEE 2004) © sin (4) (A) sin“(n) (0) tan") 120. The image formed by an objective of a compound microscope is (AIEEE 2003) (A) virtual and giminished (B)real and diminished (C)real and enlarged (0) virtual adn enlarged ‘Sol. 21. The earth radiates in the Infra-red region of the spectrum. The spectrum is correctly given by (AIEEE 2003) (A) Rayleigh Jeans iaw (8) Planck's taw of radiation (C) Stefan's law of radiation (0) Wien's tow Sol. 22. To get three images of a single object, one should have two plane mirrors at an angle of (AIEEE 2003) (ay60e (8) 90° (c)120" (0) 30° Sol. 123. If two mirrors are kept at 60° ta each other, then the number of images formed by them s (AIEEE 2002) ws (a6 (7 oe Sol. 24, Wavelenght of light used in an optical instrument are 2, = 4000 A and 4, = $000 A, then ratio of their respective resolving powers (corresponding to 2, and avis (A) 16: 25, Sol. (AIEEE 2002) (B)9:1 (C)4:5 (0)5:4 25.An astronomical telescope has 2 large aperture to (ATEEE 2002) (A) reduce spherical aberration (8) have high resolution (C) increase span of observation (D) have low dispersion Sol. 26. Which of the following is used in optical fibres? (ATEEE 2002) {(A) Total internal reflection (8) Scattering (C) Diffraction (D) Refraction Sob 1, An observer can see through a pin-hole the top ‘end of a thin rod of height h, placed as shown in the figure. The beaker height is 3h and its radiush. When the beaker is filled with 2 liquid up to a height 2h, he ‘can see the lower end of the rod. Then the refractive index of the liquid is (HEE-2002(Ser)) of » case (8) 572 (C) 372 (0) 3/2 ‘Sol. 2. Which one of the following spherical lenses does ‘not exhibit dispersion ? The radii of curvature of the surface of the lenses are as given in the diagrams. (Bee-2002(Se)] a" (¢ e* we or (é (0) » - JEE ADVANCED 3. Two plane mirrors A and B are aligned paralel to ‘each other, as shown inthe figure. A light ray is incident at an angle of 30° at a point just inside one end of A. The plane of incidence coincides with the plane of the figure. The maximum numberof times the ray undergoes ‘reflections (including the frst one) before itemerges outis (DEE 2002(Scr)} 20 2 (a)28 (®)30 (E32 (0) 34 Sol. ‘4. Aconvex lens of focal length 30 cm forms an image of height 2 cm for an object situated at infinity. If 3 conveave lens of focal length 20 cm is placed coaxially at a distance of 26 cm Is front of convex lens then size Image would be [JE 2003 (Scr)] (A)2Sem — (B)5.0 (C)1.25 (0) None 5. Ameniscus lens is made of a material of refractive index u,. Both its surfaces have radii of curvature R. thas two different media of refractive indices , a 1, respectively, on its two sides (see figure), Calculate its focal length for w, <1, < nye When light is incident on it as shown. (DEE 2003) nomen ms [on[o 6. White light is incident on the interface of glass and air as shown in the figure. If green light is just totally internally reflected then the emerging ray in air contains [BEE 2004 (Ser)} (8) violet, indigo, blue (O)all cloureexert green (A) yellow, orange, red (C)all colours. 7. A ray of light is incident on an equilateral glass Prism placed on a horizontal table. For minimum deviation which ofthe following is true? (EE 2004 (Scr)) (A) PQishorizontal (B) QRis horizontal (C) RS is horizontal (D) Either PQ or RSs honzontal. Sol, 1B. A point object is placed at the centre of a glass sphere of radius 6 cm and refractive index 1.5. The distance of the virtual image from the surface of the sphere is [DEE 2004 (Sez)] (A)2em — (B)4cm_—(C)6cm_—(D) 12am 9. Figure shows an irregular block of material of refractive index /2 . Aray of ight strikes the face AB {as shown in the figure. After refraction it is incident (on a surface CD of radius of curvature 0.4m and enters a medium of refractive index 1.514 to meet PQ at E- Find the distance OE upto two places of decimal. [DEE 2004] weiss D 410. An object is approaching a thin convex lens of focal length 0.3 m with a speed of 0.01 m/s. Find the magnitudes of the rates of change of position and Iateral magnification of image when the object is at 2 distance of 0.4 mfrom the lens. [EE 2004] Sol. 11, The ratio of powers of @ thin convex and thin concave lens is 3/2 and equivalent focal length of their combination is 30 cm. Then their focal lengths respectively are (ee 2005 (Scr)] (A)75, -50 (8) 75,50 (€)10, -15 (0)-75, 50 ‘Sob 12. Figure shows object 0. Final image 1 is formed after two refractions and one reflection is also shown in figure. Find the focal length of mirror. (in cm) = (DEE 2005(Ser)] (025 (15 (c)20 (ay10 Sot. 13, What will be the minimum angle of incidence such that the total internal reflection occurs on both the surfaces? (DEE 2005) Pc 14, Two identical prisms of refractive index J/g are kept as shown in the figure, A light ray strikes the first prism at face AB. Find, (DEE 2005) 8 > « c © (2) the angle of incidence, so that the emergent ray from the first prism has minimum deviation. (8) through what angle the prism DCE should be rotated ‘about C so that the final emergent ray also has minimum” deviation. ‘Sol. 15. A point object is placed at a distance of 20 cm {from a thin plano-convex lens of focal length 15 cm, if the plane surface is silvered. The image will form at [DEE 2006) Sem A B (B) 30 cm left of AB (0) 60 em right of AB (A) 60 cm left of AB (C) 12 em left of AB 16. Graph of position of image vs position of point object from a convex lens is shown, Then, focal length of the lens ts, (DEE 2006) waz (A) 0.50 £0.05 cm (©) 5,00 £0.05cm ‘Sot (8) 0.50 + 0.10cm (0) 5.00 + 0.10cm 17. Parallel rays of light from Sun falls on a biconvex lens of focal length f and the circular image of radius r is formed on the focal plane of the lens. Then which of the following statement is correct? (A) Area of image ar directly proportional tof (B) Area of image ar directly proportional to f* (C) Intensity of image increases iff is increased. (D) If lower half of the lens is covered with black paper area of image will become half, [JEE 2006] 18. A simple telescope used to view distant objects hhas eyepiece and objective lens of focal lengths f, and f,, respectively. Then [EE 2006) Column T Column 2 (A) Intensity of light (P) Radius of aperature R received by lens (8) Angular magnification (C) Length of telescope (() Sharpness of image Sol. (Q) Dispersion of lens. (R) focal length ff, (S) spherical aberration 19. A ray of light travelling in water is incident on Its ‘surface open to air. The angle of incidence Is 0, which Is less than the enitical angle. Then there will be (288 2007] (A) only a reflected ray and no refracted ray (B) only a refracted ray and no reflected ray (C) a reflected ray and a refracted ray and the angle between them would be less than 180° - 2u (0) 2 reflected ray and a refracted ray and the angle Lsteen thers would be grestr then 180° - 29 20, Statement-1 The formula connecting u, v and f for a spherical ‘mirrors valid only for mirros whose size are very small ‘compared to their radii of curvature [EE 2007] because ‘Statement - It Laws of reflection are strictly valid for plane surfaces, ‘but not for large spherical surfaces. (A) Statement-1is True, Statement - ITs True; State- ‘ment-II is a correct explanation for Statement-I (8) Statement-1is True, Statement - IT is True; State- ment - II is NOT correct explanation for Statement-1 (C) Statement-I is True, Statement-Il is False (0) Statement-1 is False, Statement-II is True Sol. 21. Two beams of red and violet’¢olours are made to ‘pass separately through a prism (angle of the prism is {60°). In the position of minimum deviation, the angle of refraction will be~ (DEE 2008) (A) 308 forboth the colours (8) greater for the violet colour (C) greater for the red colour (D}-equal but not 30° for both the colours ‘Sol. 22. Aight beam is tavlng from Region to Repon Ivineterrigure. The reractwe index Regions and 1 ae n, 2.22 and 8, respectively. The ‘angle of incidence 0 for which the beam just misses entering Rein Vis Fpure = (Dee 2008) 23. An optical component and an object S placed along its optic axis are given in Column-1. The dis- tance between the object and the component can be varied. The properties of images are given in Column -IL Match al the properties of images from Column- TL with the appropriate components given in Column =I. Indicate your answer by darkening the appropriate bubbles the 4 x 4 matrix given inthe ORS. Columo-I ‘Column -I “wy +) (0) Reatimage (a) irtwal image (7) Magnified image (5) Image at infinity (DEE 2008) + fh at 8) © © ‘Sol. 24. A ball is dropped from a height of 20 m above the surface of water in a lake. The refractive index of water is 4/3. A fish inside the lake, in the line of fall of the ball, is looking at the ball, Atan instant, when the ball is 12.8 m above the water surface, the fish sees the speed of ball as [Take g = 10 m/s'] [JEE 2009} (A)9m/s— (B)12m/s_—(C) 16 r/s(0) 21.33 mV/s Sol, 25. A student performed the experiment of ‘determination of focal length of a concave mirror by {u-v method using an optical bench of length 1.5 meter. The focal length of the mirror used is 24 cm. The ‘maximum error in the location of the image can be 0.2 cm. The 5 sets of (u, v) values recorded by the ‘student (in cm) are (42,56) (48,48), (60,40), (66, 33) (78,39). The data set(s) that cannot come from experiment and is (are) incorrectly recorded, is (are) [DEE 2009) (2) 42.56) (8) (48,48) (€) (66.38) (0) 78.39) 26. A ray OP of monochromatic light is incident on the face AB of prism ABCD near vertex B at an incident angle of 60° (see figure). If the refractive index of the material of the prism is /3 , which of the following is (are) correct ? (A) The ray gets totally internally refiected at face CD (B) The ray comes out through face AD (C) The angle between the incident ray and the ‘emergent ray is 90° (D) The angle between the incident ray and the ‘emergent ray is 120° (ee 2010) 27. The focal length of a thin biconvex lens is 20 cm. When an object is moved from a distance of 25 cm in front of it to SO cm the magnification of its image changes trom m,,tom,, Theratio 7 is [38E2010] 287A Biconvex lens'6t focal length 15 cm is in front of ‘plane mirror. The distance between the lens and the ‘mirror Is 10 cm. A small abject 's kept at a distance of 30 em from the lens. The final image is : [JEE 2010) (A) virtual and ata distance of 16 cm from the mirror (B) real and at a distance of 16 cm from the mirror (C) virtual and at a distance of 20 cm from the mirror (D) reat and at a distance of 20 cm from the mirror ‘Sob 29. A large glass slab (1 = 5/3) of thickness 8 cm is placed over a point source of light on a plane surface. It is seen that light emerges out of the top surface of the slab from a circular area of radius R cm. What is the value of R? [DEE 2010) Sol. 30. Image of an object approaching # convex mirror af radius of curvature 20 m along it optical axis is observed to move tram 2m te “min 30 seconds. What Is the spied of the objectin km per hours ? (DEE 2010) Sol, ‘31. Two transparent media of refractive indices y, ‘and y, have a solid lens shaped transparent material Of refractive index u, between them as shown in fig- ure in Column = 11. & ray traversing these media Is. also shown in the figure. In Column I different rela- Uionships between n,,u, and n, are given. Match them to the ray diagrams shown in Column I (DEE 2010) Cotumn IT ‘Column II o ” Onn ®) wr 2 h m © w=, ® om E » O wry Sm ” m 4 |32. Water (with refractive index = 5) ina tank is 18 cn deep Oi of retactve nde eon water make Ing a convex surface of radius of curvature R= 6 cm as shown. Consider oll to act as a thin lens. An object *S'is placed 24 cm above water surface. The location of its image is at x’ em above the bottom of the tank. then 'x'ts (DEE 2011) '33.A bi-convex lens is formed with two thin plano- ‘convex lenses as shown in the figure. Refractive index 1 of the first lens is 1.5 and that of the second lens is, 1.2. Both the curved surfaces are of the same radius of curvature R = 14 cm. For this bi-convex lens, for an object distance of 40 cm, the image distance will be- (pe 2012) naL.s. nea.2 R = 14cm. (A)-280.0.cm (8)40.0cm ()21.5em (013.3 Paragraph for Question Nos. 34to 35 Most materials have the refractive index, n > 1. So, when a light ray from air enters a naturally occuring sino, ny material, then by Snell's law, ging, ny + it 15 understood that the refracted ray bends towards the normal. But it never emerges on'the same side of the normal as the incident ray. According to electromagnetism,‘the refractive index of the medium Iscgivén by the relation, "=| 5 the Speed of electromagnetic waves in vaccum, v its ‘speed in the medium, «and ,are the relative permittivity and permeability of the medium respectively, In normal materials, both «, and p, are positive, Implying positive n for the medium. When both «and y, are negative, one must choose the negative root of n. Such negative refractive index ‘materials can now be artificially prepared and are called ‘meta-materials. They exhibit significantly different optical behavior, without violating any physical laws. Since n is negative, it results in a change in the direction of propagation of the refracted light. However, similar to normal materials, the frequency of light remains unchanged upon refraction even in meta-materials. (ee 2012] 34. Choose the correct statement. (A) the speed of ight n the meta-material is v ~ « (8) The speed of light inthe meta-material is ¥" Fj {C) The speed of light in the meta-material is v = c. (0) The wavelength of the light in the meta-material a)is Given bY Ans ark, where Ayp|s the Wave length of the light in air. ‘Sol. |36. The image of an object, formed by a plano-convex lens at a distance of 8 m, behind the lens, is real and 's one-third the size of the object. The wavelength of 2 light inside the lens is 5 times the wavelength in free space. The radius of the curved surface of the lens s- (bee 20113) (yim (8)2m_—(C)3m_— (Dm. Sol. 35. For light incident from air on a meta-material, the appropriate ray diagram is es 37. Aray tight ravelngin the direction 36 + V5) plernretel| is incident on a plane mirror. After reflection, it travels slong the creation 2-3) ange of incidence ‘ (DEE 20113) (A) 30° (B) 45° (Cc) 60° (0) 75° so (0) Netarmatert 38. Aright angled prism of refractive index u, is placed im a rectangular block of refractive index p,. which is surrounded by a medium of refractive index ,,asshown in the figure, A ray of light ‘e’ enters the rectangular block at normal incidence. Depending upon the relationships between b,p,,and by, it takes one of the four possible paths ‘ef, ‘eg, eh" or ‘ei, » Match the paths in List I with conditions of refractive Indices in List II and select the correct answer using the codes given below the lists: [38E 20113] Ust r Ust xr Pest Hr aH Qe 9.2 nm andy.rp, (Res Swen 4 Wet <2 wand LPH, Pog Rs m 2 3 4 4 @® 1 2 4 3 © 4 1 2 3 © 2 3 4 2 [ Exercise - 1 I OBJECTIVE PROBLEMS (JEE MAIN) 2B 2 C eR A & D KS A B& COM A 2 8 & A 0 B8 14 8B 12 B 13 C mm A i C16 =D 4% Bo A, Bk 22 8 2. D mw A 2. A 2% C 2% 0 mm D 2 8 3. Db MB 3c Cl Bl A 3A COA Ca a. Cc 4% BO 4% DM BAC SOD SL A SZ A SR A SD SS. DO SK OC 5 A 58 A Sf Bl OO BC BOC, 4 0 & Cc 6 B 67 A 6&8 8B 6 B mM Cc 7. 0 7 D7 Dm A [ Exercis: I JEE ADVANCED - OBJECTIVE woB 2 Cc oR C 3 8 6B on B a 8B &® A wm Cc 2A ec mB 15 BC 16 BD IR 9.98 2. Dow A 2 A 2 Bo mB wc 2. Bm 2% 8 3 B MC 3A MH AB 3S. BC 3. BD 3% A 3B A @ 0 4 C4 8 a ee oe 47K 4B Ok 508) SL AC sy oC 34 BCD SS. 8 56 OC 5% CSA SOA a. A 62 Bo 6 Cn ee oA 6. AC 7. BCD mc 7m COA % A 7% C m7 B 7. 8 7% AB 8 AC 02 AB 83. AB Bh A as. Cc 88. Bo O87 OB oo 8 9. A 91. D 92 D 9% D o A % A 97 8 OB 9. C 100 A 101 B 103. 8 104 A = 105. BD 106, AC 107, AC 108, BCD 240.8,0 111, ABC 112, AD 3. A 115. A 17.8 MB. BC 119, A,B,C 120 (i) B,D;(li)A,8,C,0; (ii) A.B,0; (iv) D [ Exercise - I (JEE ADVANCED) 1. 120° anticlockwise and 240° clockwise. 2. 30° dockwise 3.60° 4.40° 'S. Mirror should be placed on the path of the rays at an / of 78° or 12° to the horizontal. 6. Positioin of image = (1 cos 60°, -1 sin 60°), Velocity of image = 1 cos 60° j , + 1 sin 60° jm/s 7.(4, 0, 0) 8.1.23m 9. 1600m; 32cm 10,75cm —11.1000mm/s 12. infirmtely large. 23. 1.95m 14, (1) Real abject, Vitual image, (i) smaller, larger 15, Virtual object, Real image 16. larger, smaller mS Real, 2f, infinity : real 2F, cr sneee 18, (2) 40 em/s oppsite to the velocity object, (b) 20 cm/s opposite along the velocity of object. 19.600 20,20m —-24.80m/s 22/2 23.42em 24, 2410800 28.300n 26,25cn —27.35.cm, Shi) = Sam. 2B. 45° 30. sn"{ 5} siazem 32. (Se) as 35.h=/ 3 7 (37, 90° (38. 37°, This deviation is not minimum. (3 . 40, 28°=55-20n"(3}—00" an. cya se, «iy aarase 43S 45. (i) sir" [ge ane -)| (W) 4 = sinrin sina = 72.94° 46. (i), = 600 nm, = 1.5, (ii) = sin-*(0.75) =.48,59° ‘47. 240 em away trom the seperating surface ‘48, (2) 2, (b) not possible, it will focus dose to the centre if the refractive index is large ‘49. 40 cm from pole in the medivim of refractive index 1, virtual, erect and 4 cm in size. 50. S0cm 51, 27/2 #1315 cm below the surface of water 52. 4/3.) 58/S&m 54.750M,1S0cm $5. + 24cm, £120cm 56. Converging te tga Seaton imotatem — aetson sa. swan eee 1, ann sje tt of th sor Be erate cntnn= 50.8: aan Sones nd eo conven 64.-2--6.70 65. 15cm B6.11om 67. ao B.S, 24) 57. 9/4) cm’ 70.3em — 72. 75/4em,75/2em —_72. (1) 0.2m, (Wi) 0.4m 73.1.5 ‘74. (2) 1/5 = 0.2 (b) 0.728 78. 7.2° 2. (a) 21 m/s, (b) 1 x 10° /sec 5. 6/2 om 6. 16 feet B= 3, sin-¥(1/3) 9. 15 cm towards the combination 20. +60, +4/5 11, (2) taro = & = cot wy =« (2) (c) 4.0, 1 (4) It will become parallel to 13.y= 24,(0.2¢, (i) “Emm 48.1.5 or (5 - 1) 28 sem i sseniason —aaf=GAmamurion= 6m. 21-900 Exercise - IV PREVIOUS YEAR QUESTIONS [ LEVEL - I JEE MAIN ] come wc om myeeaX 0 smo he BANANA S A me mk pee \ we af ss ES .EA JEE ADVANCED ] oe “ODEN Sk kts wo 2 8B 8 Cw 6.06 m correct upto two places of decimal. ar 10. 0.09 m/s; Magnitude of the rate of change of lateral magnification is 0.3 s wc Ca coe (2) = 60*, (b) 60° (anticlockwise) Cc 16. 0c a7 Be (AYP: (BYR; (C)R; (0) P.Q,S c mc [a A oa (A) +9,.0,7 &S, (B) +a, (C) +p, ar 8s, (0) +p,a.r&s c 3s. CO 26. A, 2. 6 we OB 6am 30. 3 (A) +P, R; (B) + QS, 5 (C)-+P, RT; (D) QS 320 2am 33, BBS CKO SSEBSERS GEOM| ETRICAL OPTICS = Exarcisz-! | SINGLE CORRECT : 7. By the laws of reflection angle of incidence = Alan refed ay meet ata pom, when] Lang he reduced aan Perpendicular distance between object & mir- ror is equal to perpendicular distance between image & mirror Fig.1 shows original condition when object dis- tance is x & mirror is at mean and fig.2 shows. final condition then mirror perform SHM of am- plitude 2 cm. | Fig.2. MI" = 01+ 00" (01) =x +x-2(x-2) U'= 4a ‘A plane mirror forms inverted image of object line perpendicular to it. 3:25 Object 8:35 Image Deviation produced by plane mirror is given by = 180-21 here | = 90 - 60 = 30° = 180-60 = 120° There is a phase change of 180° in reflection. Only a portion of incident light is reflected by mirror and rest is transmitted in mid water. SO intensity of reflected light is less than inten- sity of incident light & hence image formed is. less bright. 10. 1. 2 13, 4, 15, ‘An image is called areal image if the rays after reflection or refraction actually meet hence converging rays from real image. When rays actually meet real image is formed All the reflected rays meet at 2 point, when Produced backwards. Lateral inversion refers to inverted Image of object when kept in front of mirror, Image of HOX appears same as HOX. Perpendicular distance between object & mir- ror Is equal to perpendicular distance between image & mirror. Initially the separation between object and im- age is 200 cm. After 6s the mirror has moved 30 cm,towards the object: Hence object-mir- ‘or separation is 70 cm. So object image sepa- ration is 140m. From the following figure we can see that inci- dent & reflected ray are parallel to one an- other. cet First reflection = 3 Second reflection Third reflection = 1 Total = 7 3 By the formula for the number of image formed 360 ° 1 where 0 is angle between the mirror. No. of images = 382 -1-5 Paraxial rays are considered because they form nearly a point image of a point source. 16. Tem So diameter of the image = fa =10% (1785 17. Using mirror formula 18. positive, 19. Using mirror formula 2,222 vuTt ‘The equation is in the form of y = mx +c. On comparing we se tat taking on yans and & acai than m (ope) is 1 and # iercpton yt Figure shows a rod of infinite tength with point ‘Act distance v and B at infinity. By using mirror formula we find the image of a8 2. = CO Takingu =~ 28 f= +f a,i.2 viuvf 4,1.) vitae 2th lz ytd if meet a 291 i a Magnification Is -3 because image is real & inverted. Dy using reer formula ala 6 2077 f=-15em Hereu=-300m, — f=-15cm object is at centre of curvature ~> image will be real and of same size. By using mirror formula ustif=-t aaa = ~ve (always) vorF ala oif object virtual, image always real. 2. co 3 When object is real then image move from focus to pole. ‘Somaximum distance f = 20m. av av du ao dt a % in cppsne ot Sediv «eve. ea nage then oy rom a Taity = tele ture mage then tard me Irrespective of the type of mirror. Focal length of the mirror is R/2 which de- ends on the sphere from which the mirror is. cut out. Only concave mirror forms larger image of an object. Minimum distance between abject and image is zero when image concides with the object ‘ue., object is placed at 2. It |s a convex miror. It makes a virtual image always, "300m now Magnitiation = ft = = Busy 3 (x= 300) = 3x = 900 = x 2x = 900 x= 4500m. ‘Velocity of light varies with medium. The rela- tion between velocity & refractive index is given as mM nM Where nis refractive index & v velocity of light in medium. sit Hh MA sian "HT Wy ”. Applying Snell's law on surface of incidence | 49, sngo = in(2282) pa itos (Bee -i(sin60° J) = 190 ~ | 60+ sin = 180 | si (ae } = 180° - (60 + 30] = 90° at. Inadent angie and emergent angle will be same. ihe anghe between them is 0. ‘Shift by a glass slab of thickness t is given by (2 ee) ‘And shifts towards the dath of incident light. i= 60° Displacement = t sec r sin (i-r) = S¥2 Beat] g 2 wiser Bee Now ysinr = sini By we Batig I lights traveling from medium B and suffers TR implies iy eh ) neo a (Te) vy. = sin ly v, owe > Me™ gino = sind eM [= Be v r+ P= 909 > r= (90-F) paint =y cone 6 M, tonne i 4% | Crtical angle = si | = sin (tan) In order to find the minimum diameter to block all the light we need to find the maximum. radius of the circle formed. {For radius to be maximum 8 = 8.) => r= 3m Diameter = 6m wno= w Arey of abt intering at 908 rom rarer medium tes an angle of refaction equal to cial angle in the denser medium and critical angle isgiven by 5 9, = sin 3 3 0, = tart Equation (1) & (2) 3k pe 28 wrm > a 43. 4s. We know that formula for deviation beite-A & rtrnk tei 20 402A e=0 osA AeA Asini= psinA Because angles aresmall = i= yA For minimum deviation |... A andr, = 5 Swite-Am2(i.-F) = 38° ener we Q) a (2) Now 449 = 429 + 62 - 2r > r= 30° From (1) and (2) hn = 49° In the graph for angle of deiration v/sangleof incidence the shift in angle of incidence on right side is more than that of left side x, > x,.. Hence only one angle is sutable ©=38%, From the formula oalte-A = 50 + 40- 60= 30° Sm < 30°. 4. Using formula for relation between ,,.8.A. (Aste) re) i sina oleh \2 7" “snag 0-4) sin J:= 92% a 692 > 6, 30° Now for refraction on first surface. sini=usinr, SinAw usin V2 ior minimum deviation, =r, = A/21 A ea a 2eos 5 in = V5 sine 4 gwa A 3730 = An 608 rerightbeTroTitted the ray shoul nt su fer tin necend refraction, ence Timasimum value of is less than Cthen the ray will be always transmitted wn Yan = 60° = (Faw, Ferns mins UF shoud be minimum sin (Daw In limiting €a8€ (6, )oos = % si. (ot) =e Given angle of incidence I, Given angle of emergence I Condition for minimum devidation ise = =I, Using the given formula 6=(n=1)A and tea and for f= r= = AZ Hence 4,='r. Using the formula for refraction at sperical sur- face ae oe a naa Here n= 1 u=30cm R=420cn Paper This problem can be drawn as follows -20em R= 4200n net tend tom Bm 5 We know that P= IA@Pxt=E Hence 1a = £ inal Intensity So, Now initial Intensity Focus will not change. ‘On cutting the less parallel to its principal axis f does not changes ‘so P will not change. 100 em. = R ‘We know that on cutting the lens into two parts perpendicular to its principal axis power of the two parts will be P/2 each. Let initial ower of lens be P. ‘Then (P,), = (P,), = P/2 P= (P= (P, PP, No change in power hence no change in focal length. The rays coming from infinity parallel to principal axis and paraxial meet on focus after refraction and the rays originating from focus of the lens originate parallel to principal axis afer refraction. om | The focal length of mirror formed will be f= RQ f= 20am 10 em R= 22cm {= 11cm ve sign as concave mirror is formed] f= 20cm tr] 14 woh 2, 10-1 ti” 20" “110 110 fa" 3 kn For case 1 us-u sve-k, = fe-f toi ku, * uy For case 2 ua-u,> veku, Aili -w tate (On solving (1) & (2) 0. 1 t= du+u) From the formula big = shy =a = vl =10em All are true. 1 We know that 0c = sin”? —— ge and 1, depends on wavelength of light 7 Home 25 When iis minimum the » will be maximum 8 hence 0, willbe minimum. rlmum for valet ence 0 minimum or voilet light. From the formula ‘Appartent_depth ne Real depth —n,_, ‘Apparent depth = Real depth X= The letter which appear least raised has maximum"Apparent depth and hence it has minimum yfor glass. 1 wee for to be minimum ).should be maximum which isfor Red. Using formula 15-1 0.12 Ge 7024 1.6333- 1 = 1.6161 = 0.0172 ni 1.6333-1.6161 “Teaa7-1 9276 Disp. (n,-n,) A Ray of Red light bends minimum because it has ‘maximum 3 & minimum p.. 2 e Let A'B" be the Image of tower AB. The fost of tower coincides with foot of image. Let the ‘mirror be CD then from the given condition and from 1 CAB. tnasee 5 ae h= 60m 7:34:23 ‘The plane mirror causes latered inversion Hence we can see that the time shown by image dock will be 7 : 34: 23. Hint : Make 2 clock on the paper & look at it from the back. 1M, moves on line parallel to the mirrars so to find out where M, will be able to see image of M, we have to find the total length where M, is. visible of M, so rays originate from M, & after reflection meet at M,, By using similar triangles. We find total visible length is equal to GL + 3) = 6, stance 6L speed ~ u Hence time duration will be = 5. 200em ~~ g0em 200cm Lets assume that a width of x (cm) is visible to man then from similar triangles. ADEC~ ABC 250 _ 0 ig x= Sem We know that the component of velocity parallel to mirror remains same for image but for perpendicular ten, ‘mirror (1) iS i Now to find relative velocity vz = Vj - Vp where Vj, Is relative velocity ¥, Is velocity of image (1)8 ¥ is velocity of image (2). é iad ¥ < Vga 2 sing on performing vector subtraction. The ray will retrace its path if it falls normally ‘on the mirror i.e. at third reflection angle of Incidence Is 0. From 3 ABC 0+0+90+0= 180 30=90 = 30° We know that from formula v,,= “t=¥e where V,,= Velocity of mirror Vi = Velocity of image Vz = Velocity of object We can write velocity of image for first mirror after Ist reflection vj=2V For second reflection this velocity. becomes velocity of object. -Ve we vye-4V MW=4V Thus after rf" reflection Vr2NV By image formations a eee eR + gaa ks 3 a All the images formed by two plane. Mirror inclined to each other form images which Wie on a circle. a 1sm SS Let AB be the object whose image formed by plane mirror CD is A’B'. The portion visible to the object can be drawn as shown in the ray diagram and EF Is the length visible to him. To calculate EF : 3 AGC ~ AAA'E’ &AAGD ~ 4 ANF In AAGCB ANE InAAGD B AAA AG _ AA GO + 07 AP a4 Now 1420.8 = 0.6 = EF 12s», Let AB be the street lamp of ht 3h and CD be 13. Let AB be the boy with his eye level at E and XB’ be the image then the visible portion is ‘AH, SEID ~ SEEH" 4. 15. eee ID E'H Now we know that EE" = 2 El, ID = 0.6m & Are AHe Res EN EM = 1.2 And AMeL 2 +01 = 1.3m. Hence boy cannot see his feet. Component of velocity of object 1 to mirror follows the condition. Vp =—Veelforz.component ont] y, =11k ‘The remaining components remain same as that Of the object so V, =3i + 4] +11k Let AB be the man with his eye level at E and AB’ be the image A c x fe eH FH = Siem Hence length of mirror required is FG = 85 cm and bottom of mirror should be 80 cm or less. above the ground or else feet will not be visible. Field of view Is same for all positions of the mirror and hence spot on wall remains unaffected v. 18, te t=40 a) Convex mirror with foal length = 40 em 19, 20. us -fn-1) ful=tn-1) 2. 2. 25. v (4 [v and u should be of same sign} Incorrect statement ‘Aconcave mirror forms only virtual image for any position of real object. In convex mirror Image is not at infinity (=) Image is large and real. ‘Concave mirror such that object Is closer than Image. Mirror should be placed towards left of 1. Given = -15enf"= f= 10cm > v = +30cm tesizem Poem Using mirror formula 1 a 20° =12 28. 3. V, = -Sem/s towards right. ‘So away from the mirror. Using mirror formula Roy auf he 7 wu-A t =H -t he oe =f at -20 ~15-(-20/2) x2= . wy 150 sam u= -2Sen (20m v= “100 cm ‘lag rer formats ay, (100) at 25 * 10 = - 160i For II reflection Minimum value of 0 = 45° Using mirror formula From the data given we get v= +10,u For, u, = 30m, f,= 20cm raat. 30” 20 4.4 4 2-3 4 >" 30°20" Go 7 G0 v, = -60.em For, u, = +(60 - (10 + 30)) = +20cm f= 10cm taoa 2010 > Y= +20em Now for M, Ye Total M,= mx m= Git (+20 (-60)(+20) Head (30)(20) 1, will behave as an object for M,. Hence gy. =v au a ow at Image wl go towards right, ‘The ray in this caseis not paraxial so ray after reflections does not pass from,focus but from point © secs from C. P [cP = 2f) after solving u = -30, -10 om 30 f=60 v= +30 1260 v= +36cm Ray becomes vertical means angle of incidence = 45°. .. @with x-axis = 45° Slope = +1 ” ‘A= virtual Image of object image From Snell's Law usin =p, sine at. 43, se “ae Lae Because 81, are unit vector hence wo 8 fern son Ha & int = sine 2sint= Psi & J sine ze & inva $59 Now chek opts Apparent Depth _ siz Real Depth cy (21/2) 4 c i The object will now appear to be placed at O° which isa point between C & x for ‘mirror. So image is formed between C&O. 45. 4. 51. ro For TIR to take place 0 > C. s 4 sn30° = $ sino, geen (3) sinc= c= 45.58 For TIR to take place 0 > C. b= sine =p, sini for zero deviation u, = u, hey ky = 1 If, > n, Condition for C. sing =, sin 90° For eritical angle wa sinc fat ° “ 900 C > sins BE 3 4 me cose > Waid ws wbawoag (8) © (0) From properties of prism reCaA 1 veh e0 75-4] 230 Lsini= J sinr ( 1 asin (24) in ase Using Amr, +4, ra 30° Rhee L.sini = J sin 30° ase geo (2] »= (amc) tino, < sina 4a we w> Qe First case +020 on=0 sine, = L.sin 90 1 1 sinew » ‘ A fi+0= 5 usine,'= sine a2 se sins (vant snal2 sink = sin [ in60" 8= 309i +e-A 60 +e-30=30 e=0 Normal incidence i= 0,4, = 0,7, A= 60° 12 sinc= 0 = 5 c=a2 The incident angle is greater than critical angle 0 ray will Suffer TIR. 6 = x~ 20(in case of reflection) = 180 ~ 120 = 60° Soy SO Ne On second surface for grazing emergence Wi sine, = 1sin 90° reas kent 2 Now lor ist surface Lsini= sin 15° 67. Ore A sin2A =, sind = 2SINACOSA we sink = 2cosA ‘As in previous question F,= Oand r, = 30° By using condition for prism. We have formula sini = JZ sin 30° tease A<2C > 90° < 20 2 45°C sin 45° < sinc 1 2 Bsa ne (ua) A (clockwise) ons oie = 2° clockwise Spare == 21 = 80-40 = 1769 dockwise So total = 1760 + 2° = 178° clockwise = (15-1) 50% 25% 555 ozite-A We know that if | and e are interchanged de- viation remains same. 6=14+(i+20)-60 => 40=2i-40 = 40° (e = 60°) orsimilartyi=60 (e = 40) a. For minimum deviation Renato area ra 8 a 300 Now trom Snell's law 1sini = /3 sin30° nase 6=(e- DA > 6 ABS xp-1 A» for min deviation there are two angles of av=-2R Then the distance from the centre =R+2R=IR n= 1.33 Image is always virtual because rays goes from rarer to denser medium. R=-logn u=-x 3-3/2 “~ =o a3) 2 ues ve? n= 3/2 ned from “2% =P vou we 3B ge 2 2y veaR Now for similar triangle ABE & CDE For first reflection us-Ryvate mod sue = "i204, 4, =120v=~ Retraces path If, = 240 > v= 360 =4, For virtual image V < 0. UEHx,N, =i, = Hy R= ER oy vex R m Ha, G0) vox” oR Wu v Hencev,= +ve ‘There are 3 lenses touching each other and f=, = 10cm. Letradius = R G(R) Zin PL=P,+P,+P, } x 100 (diaptor) 2) x 100 = 1082 150 Power = 6.67 diaptor sor, (F ve3u v=+30cm 100. sited 20cm ‘convex lens horizontal (=ll- aa 2 bes 80180)” 780 ~ ~ Spe Ray retraces its path when it appears to come towards centre of curvature R=20 F= 10cm For ray to retrace its path it must fall normally u= =x, f= 30cm, v= +2 2,124 =t¥"3 > T¥lens = diverging lens (concave) = focus = ~Sem 2Lens -> converging lens (convex) ‘focus = +5am R= 40cm x=30 For auto collimation the image should be formed ‘on object so the object should be placed at centre of curvature of the equivent mirror. 101. 102. 105. 106. 2 f 1 (2 | doo Pam iy * 30 ~ 2H Dia) =F.” TO (Equivalent system is concave mirror with focal length 10 cm R = 20 cm and hence w= 20em) u=20em 107. ow) 28 1 a7 tu- nla? “1 2n- Z- n (2) ‘Now equation (1) / equation (2) 10 2(y-1) 2-H D Sy 14 (u-1) Ss _ 4 » Sonata From the previous Question Equation (1) ~ equation (2) Ry 28-10%2 _ 2810-2 _ 280 "28-10 om 1B -+convex mirror C-+plane mirror 1D -rdiverging mirror 10 —+\ 1S 109. nat 3 110. Had. 2 a. 2* image = (due to combination) Dv mage (On cutting lens inte two halves power of each section becomes P/2.on combining them again net power of system becomes P so focal length of two System (i) and (1!) s some, w+u=90 Sv=54u=36 > d= 3 (= 21.6an may be used and the object be placed at a distance less than f from the lens. B diverging mirror D ~+diverging lens, b 1 neat eos bem-n[2- 12. 13. 114, 115. 116. 47. 118. 119, 120. a2. 122, ‘A+ The image will lok like a white donkey on the photograph D +The image will be less intense compared to the case in which no such glass is used. 6=6,-6,=0 (y= DA, = Gn = A, = 0 (1154 1) = (1.727 DA, =0 > A= 3° Dispersion is not possible as only one wave- length present. crs xT Me » for red C > 45° 1, Seeo ao oh One ecm elle ltl BS The emergent beam is white. “The light inside the slab Is spilt Into different colours. A -+ have dispersion without average deviation. “shave deviation without dispersion. shave dispersion and average deviation. Convex mirror always forms virtual image of real object. Due to irregular reflection. In this al the reflected rays doesn’t meet at one point in backward. R Focal length (= R= = 8 (or poranl rays beteenble to pn of (eeecionacttneis . PASR- seco as xt ot secot So PA decreases. 123. 124, 12s. OM _ Apperent depth a Real depth: In slab shift Is independent of position of object from slab. Shift = *( ) K Real image intensity of light come the during of path. In real image rays actlally converge. The ray actually converge so at the point of real image intensity of light causes the burning of paper. w 6 = 180 - 21» angle of incidence 6 = 180-2(30) = 120° anticlockwise 6 = 180 + 2(30) = 240° clockwise. Due to turning of incident ray by 10° reflected ray also gets turned by 10° anticlockwise Due to turning of mirror by 20° ray gets tumed by 40° (20) dockwise. ‘Angle turned by reflected ray = 40-10 = 30 dockwise Deviation at As, = 180 - 21, (AC) Deviation at B 4, = 180 ~ 21, (AC) In S0AB 90 - 1, + 90-1, + 60 = 180° {41 = 60° Net deviation 8 3 + 6 = 360-2(1, +1) = 360-2 x'60= 240° Fromline BC 180+0= 240 = 60° 0 129 6 For retracing the path the ray must be incident normally on second mirror. hay dreced paral to harizon torte the minor = 12" (Clade) 158. 7a (anti clockwise) Iv i= \2e08" 60° + 1sin? 60 ay2 4 noe Wk co-ordinate of image = (4,0, 0) 10. a x 200+% 50” 180 > 18x = 1000 + Sx 1000 ax = 1000 _ 76, x= TO - 76.92 em Visible floor = 200-x =123cm Use similar traingles APMI and ABI PM New 5 Mon = 40 MI” 1A 80 ~ 20 yan = 160ern AMIQL and ACL QM Mowe Mowe. 80 MI” 1A > 80 “20 = Pa = 320.0 ZNr ON SABI and SOCI " . on At oc a8 00+ 30" 100 Let Ol = x 100 = 6000 + 20x ‘80x = 6000 = x= pcm 253 = 75cm OS = OL = 75cm we 212x018 rad/s ® 12. 13. 14, 16. v7. 18. $2 - 20 - 36red/s oy oe GX - 10 cas ecto SP 25 ~ 10 23..36 - 1000 mys Ais at 2F so A’ is also at 2f B is at f so image of Bis at =, from eq. v= = 130cm) v= -390 cm, tala from = i= 5 we get f= 97.5) Radius of curvature = 2f = 1.95m (i) Real object (u = -ve) Virtual Image (v = +ve) (11) Smaller (2 = targer so uis smaller) Larger Virtual Object (u = +ve) Realimage (v= -ve) larger smaller Real, 2¢- infinity Real, 2- 19. 2. zB u=-10.emf=-20em vias thet vas hp dav 30)’ du 2-2 8 mre length of the image = 32-26 qd 2 sti = 6(1 2) = semtowards the mior u=-30cm f= 20cm velo Hee SINIS® = Sin 30" t 1 Path level by the light = 5s = core 3x10" ‘Speed of light inice = Time taken by the light to cross the slab t2 9. 1/cosis” v7 3610 /ig, d=10+10+15 35 em on for 6 = 90 there must be TIR = 180° - 21 = 90° i= 4se (90-1) >C>eosr> sine 3 since S wa 3, > 3B sing = Ssinr rswer 3 p-3siro > 31. roe 90 + 135° + 90-1 + 90-1 = 360" sree nase =F = sin"! (1/2) = critical angle 30° ly when u > 0, y | 2r = 60° r= 30" sin60° = usin30 > w= V3 »-sin(90-0) = sin = sine = ycoso b cosec 0 =x sec r, cos? 0 7 41. ny = 1,0, = 2, A= 90°, ry = 60° 4 =30 2 tan 30° 3 A=60' n=l ny=15= 5 i= 60° 3 = si isin 60 = Fsinr, syn am 4) «ase rt yehena2s 4 3, sine em sint(2) = 37. dnivena = 60° +37-60 ‘= 37 (This is not minimum deviation.) FOF bun = 1 (2) sin 2 0, = sin’ (3) inv (3) 0.= 42 F > @. TIR takes place at AC and hence angle of deviation is 180 - 21 = 60° in3 6e2-a A nage S-30 sini = 3sin30° (3 van (3) 3) = 28i0"( 3) 60°» 38° Prism angle is very small so for small angle deviation 42. 43, 45. p= 021, =A= 30° Lsini = J sin 30° 1 sini = sin ase i-e=23 23=i+e-60° > ite = 83 From equation (1) & (2) e= 30°81 =53° 1. sin S3° = ysinr nsin (60-r) =sin 30 ‘After solving eq (3) & (4) we get u~ V2sinS3> ha (4) 1 sin = n.sin (45 ~ C) = (cose -sinc) snee sain 3 PR) v2 . n (it)ry = 08 . (i) Not deviate if, = ny 5 120, 10810" .20 108 2230" 9.25 =), 600 xm (in For 5, > 1, ne 1s (3) 30° 47, Letus=x n= 3/2 ne4/3 0 vee 2-4 a o 3/2_ 4/3 _3/2-4/3 2 4 12-1 = h, =4 cm so erect, enlarged 50. For I* Refraction 1, ==10em R, = 10cm a, =1Sa,= 1 si. uy = ~(-v + 20) = Socm Ry =-10cm nye ina 15 for 1* Refraction 4 uy = -7.5em ny 91.5,n, = 4 for 2°¢ Refraction wali, 4/3 u, = -(10 +8) = -18cm 50 apparent depth = 375-1 For 1" refraction Gene Re FR =P aR ave For 2° refraction uk Pei = Ayn, ad V=ROR=-R Sn=a/3 23 snes Bsceeo yar 1 (11) relay ay For double onsen ReR Rin 2k pe? 1 (a4 507(-w It caw 75em=R, R, = 1500, weas-nfeds +E o-ve (govanduat oposte se of tens) ‘So lens in b/w image and object. «object Is real images real Ttis a converging lens because it forms a real image. For 1* surface ny = 1)» Ry=4R wens oh mm oy, y= Rk > for 2°¢ surtace U;=%, Mae fy mye REAR 1 9 7 oR OR 22 = R ers == (0) Replace u's y+ im above formula oo 1 1 t-1,100 em = 24100- 57100 cm = +100 200m us-25an $y" 235 "20 av=im why = “4x Gem = -24em P=5051=-200m As Wis vinual Image hence v and u are of samesign Vea ot « 1a ue -20 61. Sus-15cm Let x be the distance of point source from ior for mirror v should be +(25-15)-+v = 10cm So that image for mirror is made on the focus of lens so that the rays emerge parallel from lens. 0 now for mirror d.t.2 10° =x 40 2x $9213.3 tom mir ¥ ‘The image from lens is formed at C of mirror. Hence the ray will retrace its path & final image is formed at object itself and of same size. Focus length = fm ent m= -4= v/u (+. Realimage) sau ~ Ap 172 100 (Gy) Us -12.5 cm) 1a Now “a5 eek 300 ** #125 P=100 For convex lens toa 057% > f,=#200n When both lens are used 14 o7 aia %° 120-40 100 204 ams = From eq. (2) S\t For second lens the image acts as object and ‘the object for second lens is placed at 2f hence image formed is real at distance 2f on opposite ‘side So size is same as object size = d 2-4 v7 is" 10 v= 30cm, from simiar triangle ABI and COI 34 ad=tom n -4 Lb Me Zia hs Red4emv=« R, = 75/4m R, = 75/2em R= \P+(R-0.1y =R=0.2m 2 ee from eq. (1) & (2) R=30 f= 60 1 1 2H u-0(F) spas (ayn = 298-156 = 0.20 (b) 6 = (1.68 -1.56)6 a= 0.72" 5=5,-8=0 = Gyr “2A, ~ (ya 1) A= 0 = (1.62 ~ 1) 6 ~ (1518-1) A, = 0 BA =72 = 2x7 +2x(r-d)-1d=0 os far-aF 2x2 (Coes Ge) 2 [. j+r—Ve? +a? “ener See _6srv ede?) (d-0 Ve? Yd-r- ve od?) 2 @-7)-2dVe 7 (d=rP (7 +) _ 26(6- Ve) “20d # b “nae yry Wie ov ava a Fa vy 1 Var {2) Mon > WW SEM Vad Qn Vy=-tom/sec — Wy=120m/s00 Onwsce h 10) Tense, te10m 4 misec = -=1(v, -20m: + +110" m/sec = -Sh1V, -20m/s0e) > vy =42em/sec 12 100m 5 «10/3mm/see 5¥3mmisec Mere us=1Som f=10em = V=+30em Ire Mieror ue-15em V=-300n f=-100" h=-tom h=420m tens us=15em V=+30em f=+ 100m =3cm h=-6em n 690? + (6? - 6/26 em y = 2a? -ad=y% 2a? -ad) = 1-20 10, 1 ot = 202 "5 1 - = ange > gaan (113) viignamiores ven ve t2-200n v=60 Wotch class 8 nose fta0em ve ch alesis) fe bed us-15em Vest2em t= 600m m= 45 tan (90-1) = = isin 90° = u(y) sini wt sn ay) (Get t= OF = eyes oy Hert = Se = dx 4 ye (x = RY aye (FZ (iit) when y= 1 x= 4 (4,1) (iv) 1. sin 90° = 1 sine r= 90° = will beome parallel to x-axis 2 a 4 4.ain(90-0)= Sain 3 cos 0 Sain 2 sinr= Zeoso w-x>6 f = 1026 = sin(e+0)>sine F arcana : > a tesFain (8-200%6) teeta) St poe ectoc 2 = = cota > eectoe ds (9021.25 bug * (2-20) + 20-1) 180-2A+ (1-1) 2A = 150-65" es 2A 1.75 for fint Sonn ~ Sau" (1-5-1) 6= (1.751) 2A SAS? Sp: f0r Red = (1.51 = 1)6= (1.77 - 1) #2 52 Ant. w Sia for Viket = (1.491) 6- (1.731) "2 =148Ac ! 6. 3 Met aur a V, =0.8 emeee > Relatve = 2V, 1”. o? -20F a a} «ous wvim tery Vea FE “ 2) “ (5) feisam te . a aa aa 20 t=180n ve et00n ueetoen teison vee 120 oe 20. D=18m For one magnification = 2 x= 18m=x=0.6m ait 2 Ribs ote fxs 04m Separation x= 0.6m 8) (i) For Lens > u = 90 em f= +30cm, v= +45 cm (2) for spherical Refraction us+45n,=4/3 net R=-30cm set ,itei > vy “" 90 * 45 ~ 90 =V=120em Image through plane mirror is at Mirror 40 em from the plane. u=-400m n=, 0-4/3 R=+300n PREVIOUS YEAR QUESTIONS ] fabeOg For focal length ofthe len, rita cm. 249?" 2 Now, to get back image on the film, lens has to a form image at (12-3) em = 33 em such that the glass plate will shift the image on the fim. 560 u=-5.6m 1680 c 1 Apparent shift ah = (1-2 Apparent shift produced by water, 1 Bs ‘and apparent shift produced by kerosene, an =(122}, ah, -[1- anwah, ah, | Hence, c Scattering for blue light is largest and itis polar- {zed by scattering. ‘Also for polarized light T= I, cos’® x JEE MAIN a Note: As refractive index for z > O and z < Ois different X-Y plane should be boundary between two media. Angle of incidence io varo ar cosi -| As intensity is maximum at axis, ‘4 will be maximum and speed will be minimum on the axis of the beam. beam will converge. A A It is passible when object kept at centre of curvature. samen sinr-s(90"-C)neoec = sine a sin” wy sin = Fx oe un Jp c s uu. = constant, so (c) Is the correct graph. 12.8 Power of a lens is reciprocal of its focal length. power of combined lens is PaPtP, “158 +5 = -100 D=(rl)A For blue light is greater than that,fonred light: irekens inp The situationis'shown in figure : ina, - 2 me nb, tae, = At sscohenk 15.4 Y o4222 We know ¥ 21.224 we Ta 10? «3x10? 122x5x10 20s. o. 5m =D... =5m ofl) For total internal reflection from glass-air interface, criti- c cal angle C must be less than angle of incidence tee or C< 457) 445") WA fe ps gteoeesm(!) ° & ssinn'(t}<4se ox (2) sings oe ne tiss 1 wR ont a A plano-conves lens behaves as a concave mirror if its one surface (curved) is silvered. The fays refracted from plane surface are reflected from cruved surface and again refract from plane surface therefore, inthis lens two refractions and one reflection occur Let the focal length of silvered lens is F. aa 2,4 at Where, f = focal length of lens before silvering. 1. focal length of spherical mirror 22.8 Number of mages, n = 360° Hence, from 4. () Where vs angle between mirrors. 2,2 12,26 3-3 Fe 10cm oro= 90° Again given that, size of object = size of mage 23.4 Onl Number of images, n = 262° - Where a is angle between mirrors. Thus, 060 (given) So, number of images 360 5 ° 24.0 Resolving power of afi optical instrument is in- u=-20em versely proportional tote. Hence, to get a real image, object must be placed : at a distance 20 cm on the left side of lens. np. 18.0 Resolving power ata, _ Tha particular angle of incidence for which Fe ere ower eh ot flected light is totally polarized for reflection from Resaiving power: 64; 2, air to glass;'s called the angle of polarisation (i,) 5000 (Brewster's law) ~ 4000 -**4 Accordingly, 9 = tani, 25.8 4, = tavi¢n) nis refracti ; aeA Whereis refractive index of gas. Satie Nbres work on the principle of otal nter- 20. reflection. Objective of compound microscope is a convex lens. Convex lens forms real and enlarged image when an object is placed between its focus and lens, 2A ‘After every x distance there is 2 reflection so number of reflection naa ~ "02 ” For concave hos Ferg ohn 2sem ® Ma-he Many R=R wo HRM HRM R we wen Weey MAH Roy, RR Ye that _ (ay “oR JEE ADVANCED —_] =o (2g) (tage) v2 UR Ry ‘i case of minimum deviation r, =F, = A/2 For this QR is honzontal. LS ie v6 Ge ve Gem, z= 1514, m2 1sin45= V3 snr ee a | dm +f au 30 + 0.01 «4 = 0.01 « 100 (= & “(40-307 a 4 Retraction ng =1; m= 44 6=33.25 @-3asn25-2404 conn D ) 7k Refraction b= (4-18) 9g: my -4 4-25an 24 275-460 5, 38 . f= 183m = 20cm “lé fi Ctormatace an sno 2. rg on as Cforintertace | & sne- sceor minimum angie = 60° (0) for 5 >= 30" 1. sin i= ¥3 sin 30" = i= 60" (b) actas a slab 16. 17. i = f=Sem om, Av=0.1.em = M=0.05en eat (A)-P (B)-R: (C)-R; (0)-P.0.S c c han oe N= a3 lust mites means we are taking about critcal angle in mediurr 3. So angle of refraction in medium 4 is 90". nesna-("3 Jans =a (A) +PQRS : (8) +0: (C)4P.ARS: OP.ARS Vg = V2 10> 7.2 = 12m/s () (42, 56)-> Z2 - = F (comecty recorded) a 4B a 1 (i (66,38) => ~ Fe - -i 1 65.8 37.8 24.008 6078.39) > -,-E = Z i) (48,48) (correct) ot 24 (ncorect) 1.sin 60° = 3 sinr o=4s* ‘So angle of mcidence at CD is 45° Creal Angle for the surface evs (3) tstevet Cah tre 44 STENT Aw te: eae for case (1) = 206m, case-2 30 on 14 4 y 100 "30750 29" 3 Mgg = 2300..-2, Mas <4 Mang SOT 3NSO 3° Msp = 2/3 ~ Meg (Object is placed at the 2 of lens. Image formed by wit behave 0s objec for plane miror(vrtua object fraty) 32. 0's ‘speed of abject = 3 kmite. trom laws of Retraction n, sni=n, sine Denser to Rarer + Away from normal Rarer to Denser -> Towards the normal. ‘After Critical angle all the rays are Reflected {and so No transmition occurs.

You might also like